Download as pdf or txt
Download as pdf or txt
You are on page 1of 103

XI MODULE - II

CHEMICAL PERIODICITY &


CHEMICAL BONDING

Modus Operandi for Practice


(Suggested Ideal Approach)

Step – I CatalyseR 's Practice Sheets & NCERT


{while Chapter is running in Class-Room}
(Mandatory)

Step – II
CatalyseR 's Module
(Mandatory)

Step – III Previous Years’ JEE Subjective & Objective Questions,


R C Mukherjee solved and unsolved questions
(Mandatory)

Step – IV After Completion of Step – III, if time permits, students can solve
questions from previous year’s INCHO, NCHO, KVPY papers,
(Optional) Physical chemistry books by Atkins, Bruce H. Mahan

This Study Package is Prepared by


d`fr
o f Ca talyseR
on te nt Wing
C
ELECTROSTATICS 1

CHEMICAL PERIODICITY
INDEX

 CONCEPTS IN BRIEF (CHEMICAL PERIODICITY) 1 - 11


 SOLVED EXAMPLES 12 - 14
 EXERCISE # 01 OBJECTIVE EXERCISE (LEVEL # 01) 15 - 19
 EXERCISE # 02 OBJECTIVE EXERCISE (LEVEL # 02) 20 - 23
 EXERCISE # 03 OBJECTIVE EXERCISE (LEVEL # 03) 24 - 31
 EXERCISE # 04 SUBJECTIVE EXERCISE (LEVEL # 01) 32
 EXERCISE # 05 SUBJECTIVE EXERCISE (LEVEL # 02) 33 - 34
 EXERCISE # 06 KVPY, OLYMPIADS QUESTIONS 35
 EXERCISE # 07 JEE (MAIN) CORNER 36 - 37
 EXERCISE # 08 JEE (ADVANCED) CORNER 38
 ANSWER KEYS 43 – 45

WEIGHTAGE OF ‘CHEMICAL PERIODICITY’ IN JEE (MAIN & ADVANCED) in Last Three Years

JEE (MAIN) Formely known as AIEEE

MARK /
YEAR No. Of Qs.
CHEMISTRY TOTAL MARKS
2015 1 4/120
2016 1 4/120
2017 1 4/120

JEE (ADVANCED)

MARK /
YEAR No. Of Qs.
CHEMISTRY TOTAL MARKS
2015 0 0/168
2016 1 3/124
2017 1 4/122

Nothing is impossible, the word itself says ‘I’m possible’!


CHEMICAL PERIODICITY 1

CHEMICAL PERIODICITY
INTRODUCTION
Periodic table helps us to undertake a systematic study of the various elements found in nature
without which it would have been impossible for us to study all the elements in the table.
Many attempts were made to classify the known elements from time to time. These are :
(i) Lavoisier classification (ii) Dobereiners Triad law
(iii) Newlands Octave law (iv) Lothar Meyer's curve
(v) Mendeleev Periodic law (vi) Mosley experiment
(vii) Modified mendeleev periodic table (viii) Modern periodic law

DOBEREINER'S TRIADS, 1829


In the triads of elements the atomic weight of the middle element was the arithmetic mean of the
atomic weights of the other two. Some of the triads are as under:
Li Na K
7 23 39
Ca Sr Ba
40 88 137

NEWLAND'S LAW OF OCTAVES, 1864


If the elements are arranged in order of their increasing atomic weights, every eighth element had
similar properties to the first one like the first and eighth note in music.
Li Be B C N O F
Na Mg Al Si P S Cl
K Ca

LOTHAR MEYER'S ATOMIC VOLUME CURVE 1869


i) Lothar Meyer plotted a graph between atomic weight and atomic volume (i.e. atomic
weight in solid state/density)
ii) Elements with similar properties occupied similar positions on the graph.
iii) Strong electropositive elements of IA except Li i.e. Na, K, Rb, Cs etc. occupied the top
position on the graph.
iv) IIA group elements Be, Mg, Ca, Sr, Ba etc. occupied the positions on the descending part
of the graph.
v) Inert gases except He occupied the positions on the ascending part of the graph.
vi) Halogens also occupied the ascending part of the graph.

MENDELEEF’S PERIODIC LAW


i) The physical and chemical properties of elements are a periodic function of their atomic
weights.
ii) If the elements are arranged in order of their increasing atomic weights, after a regular
interval elements with similar properties are repeated

Nothing is impossible, the word itself says ‘I’m possible’!


2 CHEMICAL PERIODICITY

MENDELEEF'S PERIODIC TABLE


The table is divided into nine vertical columns called groups and seven horizontal rows called
periods.

MODERN PERIODIC LAW (MOSELEY’S PERIODIC LAW)


Physical and chemical properties of elements are the periodic functions of their atomic number.If
the elements are arranged in order of their increasing atomic number, after a regular interval,
element with similar properties are repeated.

PERIODICITY
The repetition of the properties of elements after regular intervals when the elements are
arranged in the order of increasing atomic number is called periodicity.

CAUSE OF PERIODICTY:
The periodic repetition of the properties of the elements is due to the recurrence of similar
valence shell electronic configuration after certain regular intervals.
The modern periodic table consits of horizontal rows (periods) and vertical column (groups)

Discovery of element 104, 105 and 112 has recently been reported
10
[(n–2)f1–14(n–1)d0– ns2]

(4f1–145d0–106s2)

(5f1–146d0–107s2)

CatalyseR Eduventures (India) Pvt. Ltd.


CHEMICAL PERIODICITY 3

CLASSIFICATION OF ELEMENTS ON THE BASIS OF THEIR ELECTRONIC CONFIGURATION


i) s-Block elements
a) These are IA and IIA i.e. 1 and 2 group elements.
b) In these elements last electron fills in the s-orbital.
1-2
c) Electronic configuration of valence shell is ns ( n = 1 to 7).
ii) p – block elements
a) These constitute the groups IIIA to VIIA and zero group i.e. groups 13 to 18 of the
periodic table.
b) The last electron fills in the p-orbital of valence shell.
c) The electronic configuration of the valence shell is ns2np1-6 ( n = 2 to 7).
iii) d-Block Elements
a) These constitute IIIB to VIIB, VIII, IB and IIB i.e, 3 to 12 groups of the periodic table.
b) The last electrons fills in ( n – 1)d orbital.
c) The outermost electronic configuration is (n-1)d1-10 ns1-2 (n = 4 to 7).
d) Their two outermost shell are incomplete.
iv) -Block Elements
a) These are mainly related to IIIB i.e. group 3 of the periodic table
b) There are two series of -block elements as under
4 series – Lanthanides – 14 Elements
Ce (58) to Lu (71)
5 series – Actinides – 14 Elements
Th (90) to Lw (103)
c) The last electron fills in ( n – 2) -orbital
d) Their outermost electronic configuration is (n-2)1-14 (n-1)s2 (n-1)p6 (n-1)d0-1ns2
(n = 6 and 7).
e) Their three outermost shell are incomplete.

SOME COMMONLY USED TERMS :


1. Noble Gases : Element of group 18 are called noble gases. These are also called as inert gases
because their outermost ns and np orbitals are completely filled (except He and 1s2) and these
gases are non-reactive in nature under ordinary conditions.
2. Representative elements : All the s and p block elements are known as representative elements
except zero group.
3. Transition elements : All the d-block elements (except IIB group) are called transition element. It
comprises into 4th, 5th, 6th and 7th period. They lie between s and p block elements.
4. Inner transition elements : All the f-block elements or 4f and 5f block elements are called inner
transition element. Total number of these elements is 28. They lie in IIIB and placed at the bottom
of periodic table.
5. Typical elements : Third period elements are known as typical elements. These include Na, Mg,
Al, Si, P, S & Cl.

Nothing is impossible, the word itself says ‘I’m possible’!


4 CHEMICAL PERIODICITY

DIAGONAL RELATIONSHIP

On moving diagonally across the periodic table, elements show certain similarities. These are called
diagonal relationships, which are weaker than the similarities within a group, but quite pronounced in the
following pairs.
Li Be B C

Na Mg Al Si
On moving across a period the charge on the ions increases and the size decreases. On moving down a
group ionic size increases. On moving diagonally these two effects cancel each other and the properties
especially polarizing power. (Refer Fajan’s rules Lesson 4) become similar.

BRIDGE ELEMENT : -

Third period elements are called ‘Bridge Elements’. As the division between two subgroups A & B
starts from these elements.

NOMENCLATURE OF THE ELEMENT WITH ATOMIC NUMBER > 100 (IUPAC):


The names are derived by using roots for the three digits in the atomic number of the element
and adding the ending –ium. The roots for the number are
Digit Name Abbreviation
0 nil n
1 un u
2 bi b
3 tri t
4 quad q
5 pent p
6 hex h
7 sept s
8 oct o
9 enn e

Thus element with atomic number 109 will be named as Unnilennium (Une)

PREDICTION OF PERIOD, GROUP AND BLOCK :

 Peiod of an element corresponds to the principal quantum number of the valence shell

 The block of an element corresponds to the type of subshell which receives the last electron

CatalyseR Eduventures (India) Pvt. Ltd.


CHEMICAL PERIODICITY 5

 The group is predicted from the number of electrons in the valence shell or/and penultimate shell
as follows.
(A) For s-block elements
Group number = the no. of valence electrons

(B) For p-block elements


Group number = 10 + no. of valence electrons
(C) For d-block elements
Group number = no. of electrons in (n – 1) d sub shell + no. of electrons in valence shell.

THE PERIODICITY OF ATOMIC PROPERTIES

EFFECTIVE NUCLEAR CHARGE AND SHIELDING EFFECT


i) In a multi-electron atom, the effect of nuclear charge experienced by the outermost electron is
less than the theoretical value of the nuclear charge (Z).
ii) If the nuclear charge of an atom is Z, effective nuclear charge is Zeff and shielding constant is 
then according to Slater
Zeff = Z –
Slater’s Rule :
(A) For calculating  on a (s or p) block (other than on 1s)
Rule – 1: Each (ns, nP) electron contribute to a screening factor of 0.35.
Rule – 2: Each (n – 1)th shell electron contribute to a screening factor of 0.85.
Rule – 3: Each (n – 2)nd and deeper shell electron contribute to a screening factor of 1.
[On 1s, the screening factor due to other electron is taken as 0.3]

(B) For calculating  on (d or f) block.


Rule – 1: Each screening causing electron (d or f) of same shell has factor of 0.35.
Rule – 2: Each electron other than Rule – 1 have screening factor of 1.

General Trend
Along a period, size decrease [‘n’ constant, Zeff  ]
Along a group, size increase [‘n’ increasing, Z eff constant]

ATOMIC RADIUS:
Problem in calculating actual size of atom and hence distance between nuclei is calculated giving
rise to three types of radii for atoms.

(A) Covalent radius :


It is one-half of the distance between the centres of two nuclei (of
like atoms) bonded by a single covalent bond. Covalent radius is
generally used for non-metals.

Nothing is impossible, the word itself says ‘I’m possible’!


6 CHEMICAL PERIODICITY

d A A
(A) For Homoatomic molecules d A A  rA  rA or 2rA so, rA 
2
 (B) For hetrodiatomic molecules in which electronegativity remains approximately same.
d A B  rA  rB
 For heteronuclear diatomic molecule, A–B, where difference between the electronegativity values
of atom A and atom B is relatively larger,
d A B  rA  rB  0.09  X A  X B  (In this formula unit of length is taken Å)

where X A and X B are electronegativity values of A and B respectively.

(B) Van der Waals radius (Collision radius) :

It is one-half of the internuclear distance between two adjacent atoms in two nearest
neighbouring molecules of the substance in solid state.

(C) Metallic radius (Crystal radius) :

It is one-half of the distance between the nuclei of two adjacent metal atoms in the metallic crystal
lattice.

rcovalent < rcrystal < rvander Walls

Variation in a period: On moving left to right due to increased nuclear charge the size
decreases.
Variation in a group: On moving top to bottom due to extra addition of a shell the size increases.

CatalyseR Eduventures (India) Pvt. Ltd.


CHEMICAL PERIODICITY 7

FACTORS AFFECTING ATOMIC SIZE :


(A) ‘n’ increase size increases
(B) Zeff increase size decrease [Zeff = Z – ]
(C) Type of measurement of radii.
Calculation of Z eff

En 2 E  n2
Z eff  where E is I.E. in kJ/mole; Z eff  E is I.E. in eV per atom
1312 13.6
or Calculated by Slater’s rule.

Exceptions :
(1) Noble gases have largest atomic sizes [Vander waal radii]. However, their covalent radii are
smaller e.g. Xe.
(2) Size of Ga and Al are same, [Zeff increasing]
(3) Size of Hf & Zr are same (lanthanide contraction)

IONIC RADIUS : A cation is smaller than parent atom . An anion is larger than parent atom.

ISOELECTRONIC SPECIES [Size depends upon Z, more Z less size]


The species containing the same number of electrons but differ in the magnitude of their
nuclear charges are called as isoelectronic species. For example, N3– , O2–, F–, Ne, Na+ ,
Mg2+ and Al3+ are all isoelectronic species with same number of electrons (i.e 10) but
different nuclear charges of +7, +8, +9, +10, +11, +12 and +13 respectively.
For Example,
Al3+ Mg2+ Na+ F– O2– N3–

Ionic radii increase

As effective nuclear charge decrease.

IONISATION ENERGY :
Amount of energy required to remove the most loosely bounded electron from an isolated
gaseous atom from its ground state electronic configuration.
Units : kJ mol–1, k cal mol–1, eV per atom.
Ionisation is endothermic (endoergic) i.e. requires energy hence Hionsation is +ve

M + Energy (IE1)  M+ + e–

M  M+ + e– H = IE1
M+  M+2 + e– H = IE2
M+2  M+3 + e– H = IE3
IE3 > IE2 > IE1 always

Nothing is impossible, the word itself says ‘I’m possible’!


8 CHEMICAL PERIODICITY

FACTORS AFFECTING IONISATION ENERGY :

(1) Atomic size : Varies inversely


(2) Screening effect : varies inversely
(3) Nuclear charge (Z) : varies directly
(4) Special electronic configuration of outermost electron (half filled / fully filled)
(5) Type of orbital involved in Ionisation :s > p > d > f.
Half fillness and full fillness of inner orbitals. [affects d block and f block trends]

General Trend: Along period I.E. increases [with some exception] [Zeff ]

Along a group I.E. decrease [Zeff constant, n ]

Exception :

(1) Along a period, half filled and fully filled have higher I.E.
e.g. Be > B and N > O.
(2) along a group, Ga > Al

PROPERTIES AFFECTED BY IONISATION ENERGY:

(1) Metallic character : Varies inversely


(2) Reducing power : Varies inversely
(3) Reactivity (Metals) : Varies inversely
(4) Determination of oxidation state (Tendency to stay in which state A1 , A2 or A3 )

a) If the difference between two consecutive I.P.'s of an element is 16 eV or more, the lower
oxidation state is stable.

b) If the difference between two consecutive I.P.s. of an element is 11.0 eV or less, the higher
oxidation state is stable.

ELECTRON AFFINITY :

Amount of energy released when an electron is added to an isolated gaseous atom.


Units : k J mol 1 , k cal mol 1 and eV per atom.
e.g. Cl(g) + e–  Cl– + energy

Addition of electron results in release of energy in most of the cases but 2nd E. A. is always
energy required. The sum of EA1 & EA2 is energy required.

E A  1/Atomic Size  Zeff . Cl has the highest E.A.

CatalyseR Eduventures (India) Pvt. Ltd.


CHEMICAL PERIODICITY 9

ELECTRON GAIN ENTHALPY : (EGE)

When expressed in terms of enthalpy change (Heg) then it is termed as EGE Remember that

H = – ve for exothermic change.


For EA1, energy is released    Heg1 = – ve (Not always)
For EA2, onwards is required    Heg2 = + ve (always)

Example
Heg  ve
M(g) + e–  M–(g) energy released
E . A.   ve

Heg 2   ve
M–(g) + e–  M2–(g) energy absorbed
E . A.2  ve

FACTORS AFFECTING ELECTRON AFFINITY :

(1) Atomic size : varies inversely


(2) Nuclear change : varies directly
(3) Stable electronic configuration i.e. half filled and fully filled shells have smaller or negative EA.

General Trend :
Along a period, electron affinity increases [with exception] as Zeff .

Along a group, electron affinity decreases after 3rd period. Between 2nd and 3rd period in p block
electron affinity of 2nd period is lesser due to high electron density.

Exception :

(1) A fully filled and half filled which have low values or even sometimes energy is required rather
than getting released.
(2) 2nd period has lower value than 3rd owing to repulsion between electrons.

ELECTRO NEGATIVITY

It may be defined as the tendency of an atom to attract the shared pair of electrons towards itself
in a covalently bound molecule.
F has highest electronegativity in periodic table.
Decreasing order  F > O > Cl = N > Br > S = C > I > H.

Nothing is impossible, the word itself says ‘I’m possible’!


10 CHEMICAL PERIODICITY

Different Scales of Measurement of Electro Negativity


Pauling Scale: XA  XB = 0.208   in kcal/mol

XA  XB = 0.102   in kJ/mol
  = EA  B  (EA  A x EB  B)1/2
IE  EA
Mulliken's Scale : XA =  eV 
2
Mulliken's values of E N are about 2.8 times as large as Pauling.
XM
i.e. Xp 
2.8
0.359 Zeff
AllredRochows : XA = + 0.744
r2
r = covalent radius of atom in Å
Zeff = Effective nuclear charge on periphery

FACTOR AFFECTING ELECTRO NEGATIVITY :

(1) Nuclear attraction : varies directly


(2) Atomic radius : varies inversely
(3) Charge on ions : More positive charge more electronegativity and more –ve charge less
electronegativity.
(4) Hybridisation : to be discussed later in bonding. (With the increase in s character
electronegativity increases)

General Trends : Along a period, electronegativity increases


Along a group, electronegativity decreases
Exceptions : None noteworthy.

APPLICATIONS :

(1) % ionic character varies directly.


Henny smith formula % ionic character = 16 + 3.5 2
 = | XA – XB|
Bond to be 50% ionic  = 2.1

(2) Strength of bond varies directly.

(3) B.L. : varies inversely.

(4) Nature of oxides – Basic if X O  X E  2.3 , Acidic if X O  X E  2.3 . where X O and X E are
the electronegativity of oxygen & element respetively.

CatalyseR Eduventures (India) Pvt. Ltd.


CHEMICAL PERIODICITY 11

(5) Nature of hydroxide.


A hydroxide MOH of an element M may ionise in two ways in water.
M–O–H + H2O MO– + H3O+ (when XO  X H  XO  XM )
M–O–H +H2O MOH2+ + OH– (when XO  X M  XO  X H )
where x0, xH and xM are the electronegativities of oxygen, hydrogen and element respectively.

(6) Nature of hydride.

MISCELLANEOUS CHEMICAL PROPERTIES :

1. Periodicity of hydra acids :


(A) Acidic character of hydra acid increases from left to right in a period.
(B) Acidic character of hydra acid increases from top to bottom in a group.

2. Periodicity of oxy acids :


(A) Acidic character of oxy acid increases from left to right in a period.
(B) Acidic character of oxy acid decreases from top to bottom in a group.

3. Periodicity of nature of oxide :


(A) On moving from left to right in a period acidic nature of oxide generally increases.
e.g. CO2 < P2O3 < SO2 < ClO2
(B) On moving from top to bottom in a group acidic nature of oxide generally decreases.

4. Solubility of salt in water :


(A) Hydration energy decreases along a group.
(B) Lattice energy decreases along a group.



Nothing is impossible, the word itself says ‘I’m possible’!


12 CHEMICAL PERIODICITY

SOLVED EXAMPLES

1. Why is the atomic radius of oxygen slightly more than that of nitrogen?
Solution: Nitrogen atom has a half-filled sub-shell 2p3; the next electron in oxygen enters an
4
occupied p-orbital (2p ); so the 2pz orbital has a pair of electrons. The resulting repulsion
between this pair of electrons more than offsets the attraction of the increased nuclear
charge. This can be seen from the values of the atomic radii given in the table.
2. Which one in larger size and why?
+ 2–- + –
a) Na or Na ; (b) S or S ; (c) K or Cl

Solution: a) When an electron is removed from an atom, the effective nuclear charge
increases. Hence, the electron cloud shrinks.
 Na  Na+
b) When one or more electrons are added to a neutral atom, the nuclear charge has
to attract the additional electrons also. Hence, effective nuclear charge
decreases.
S2–  S
c) K+ and Cl– are isoelectronic, that is, they possess 18 electrons. In the case of K+,
there are 19P attracking the 18 electrons while in Cl– there are only 17P
attracting the 18 electrons. Hence, effective nuclear charge is greater in K+.
 Cl–  K+

3. Nitrogen has higher ionisation energy than oxygen – explain.


Solution: In case of nitrogen, we see that it has a stable half filled shell 1s22s22p3. On the other
hand oxygen has neither a half filled nor a completely filled shell (1s22p22p4).
Consequently N must have higher I.P. value than oxygen. However, although the first
ionisation potential of N is greater than that of O, the second I.P. of N is much less than
that of oxygen. This is because, after the removal of 1 electron from oxygen atom it
3
attains stable 2p configuration (half filled shell).

4. Why electron affinity of nitrogen is less than that of carbon?


Solution: It is due to the comparatively stable half filled configuration (np3) of nitrogen and
3
phosphorus and the tendency to acquire the stable np configuration by the gain of one
electron in carbon and silicon (np)2.

5. Generally second electron affinity is endothermic – explain


– – –
Solution: When the anion A forms a bi negative ion A , it will not accept the second electron
easily due to anion-electron repulsion. Hence work is to be done on the system to force
the electron to enter inside the electronic structure. Consequently the second electron
affinity must be endothermic (H is positive).

CatalyseR Eduventures (India) Pvt. Ltd.


CHEMICAL PERIODICITY 13

6. The EA of Mg is negative while that of Cl is positive. Account for the observation.


Solution: Mg has the electronic configuration 1s22s22p63s2. It has a tendency to lose electrons in
order to attain a stable octet configuration. When an electron is introduced into its
outermost shell, energy has to be supplied, i.e. it is an endothermic process. EA is
assigned a negative value. Chlorine has the electronic configuration 1s22s22p63s23p5. It
readily accepts an electron. Hence, the electronation process is exothermic and its EA is
positive.

7. Electronegativity of Sn in SnCl2 is different from that of Sn in SnCl4 - explain


Solution: The electronegativity values increases as the oxidation state of the atom increases. So
the attractive power of Sn(IV) in SnCl4 for electron is much greater that that of Sn(II) in
SnCl2.

8. Mg has a higher melting point of a metal than Na. Explain


Solution: Melting point of a metal depends on the metallic bond strength. Mg has stronger metallic
bonds due to
a) Its smaller size
b) Greater number of valence electrons involved in metallic bonding.

9. Among fluorine – fluorine bond and chlorine – chlorine bond, which is more
stronger and why?
Solution: In Cl—Cl bond, a filled p-orbital of chlorine can overlap with a suitable vacant d-orbital of
adjacent chlorine thereby introducing some double bond character. Thus the bond
strength increases. This is not possible in fluorine as it has got no vacant d-orbital.

10. 2nd electron affinity for halogens is zero – explain


Solution: After addition of one electron in halogen atom formation of halide ion takes place which
has fully filled configuration (ns2np6) and thus possesses no tendency to gain one more
electron.

11. NaOH behaves as a base while Zn(OH)2 is amphoteric why?


Solution: In NaOH the bond electronegativity difference between Na and oxygen is greater than

between H and O and therefore it is the Na–O bond that breaks releasing OH . But in
case of Zn—O—H bond the difference of electronegativity of Zn—O and O—H are
almost same. So there is equal probability that the bond breaks in both ways leading to
an amphoteric behavior

12. LiCl has a lower melting point than NaCl. Explain


Solution: Since LiCl is smaller in size than Na, it has a higher I.E. Therefore, it has a greater
tendency to form covalent compounds. LiCl, being a covalent compound, has a lower
melting point than NaCl which is ionic in nature.

Nothing is impossible, the word itself says ‘I’m possible’!


14 CHEMICAL PERIODICITY

13. Why does xenon react with fluorine whereas neon does not?
Solution: Xe has a lower ionisation energy than Ne. The valence electrons in Xe
(n = 5) are much farther from the nucleus than those of Ne (n=2) and much less tightly
held by the nucleus; they are more willing to be shared that those in neon. Also xenon
has empty 5d orbital which can help to accommodate the bonding pairs of electrons,
while neon has all the valence orbitals filled.

14. The electron affinity of chlorine is +37eV. How much energy in kJ is released when
3.55 g of chlorine is converted completely into Cl– ion in the gaseous state. (IeV =
96.45 kJ / mole)
Solution: Cl(g) + e–  Cl– (g) + 37 eV
Energy released when 1 mole (35.5 g) of chlorine atoms change completely into Cl–1(g)
= 37  96.49 kJ / mol = 357.01 kJ
Energy released when 3.55 g of chlorine atoms change completely into Cl– (g)
357.01
=  3.55 = 35.701 Kj
35.5

15: Compare qualitatively the first and second I.P. of Cu & Zn. Explain the observation.
Solution: First I.P. of Cu is less than that of Zn because removal of one electron from 4s orbital of
Cu leaves completely filled orbitals and in that of Zn, a completely filled 4s orbital is
converted to half-filled orbital. But in case of 2nd I.P. the case is reversed as Cu+ gives an
incompletely filled orbital (3d9) and Zn+ gives completely filled (3d10) orbital.

16: The electron affinity of sulfur is greater than oxygen. Why?


Solution: This is because of smaller size of oxygen due to which it has got higher charge density
and thus electronic repulsion increases as it takes electron. So its E.A. is less than
sulphur.

17: In s & p block elements the oxidation state change by 2 units but in transition
elements it changes in units of 1. Explain.
Solution: This is due to a pair of electrons remaining paired in –ous form and becoming unpaired in
–ic form (e.g. Sn, P, Te etc.]. But in transition metals different no. of d-electrons may take
part in bonding [e.g. Fe2+, Fe3+ & Cu+, Cu2+ etc.].

















CatalyseR Eduventures (India) Pvt. Ltd.


CHEMICAL PERIODICITY 15

EXERCISE # 01 OBJECTIVE EXERCISE (LEVEL # 01)


BASED ON IDENTIFICATION OF GROUP & PERIOD

1. An element has electronic configuration 1s2 2s2 2p6 3s2 3p4 . Predict their period, group and block
(A) Period = 3rd, block = p, group = 16 (B) Period = 5th, block = s, group = 1
(C) Period = 3rd, block = p, group = 10 (D) Period = 4th, block = d, group = 12

2. The most predominantly ionic compounds will be obtained from the combination of elements
belonging to
(A) 1 and 7 groups (B) 2 and 6 groups (C) 3 and 5 groups (D) 0 and 7 groups

3. An element M has an atomic mass 19 and atomic number 9. Its ion is represented by
(A) M (B) M (C) M2  (D) M2 

4. The elements indicating following atomic numbers belong to same group


(A) 11 and 37 (B) 19 and 15 (C) 39 and 88 (D) None of these

5. Which of the following atoms has the largest atomic radius?


(A) Cs (B) Ba (C) Pb (D) Cu

BASED ON LONG FORM OF PERIODIC TABLE

6. The nitride ion in lithium nitride is composed of


(A) 7P  7e (B) 10P  7e
(C) 7P  10e (D) 10P  10e
7. Which set has the same number of unpaired electrons in their ground state
(A) Cl ,Fe3  ,Cr 3  (B) Na , Mg2 , Al
(C) Na,P, Cl (D) N,P, V

8. The elements with atomic numbers 9, 17, 35, 53, 85 are all
(A) Noble gases (B) Halogens
(C) Heavy metals (D) Light metals

9. Which of the properties remains unchanged on descending a group in the periodic table
(A) Atomic size (B) Density
(C) Valence electrons (D) Metallic character

10. The cause of periodicity of properties is


(A) Increasing atomic radius
(B) Increasing atomic weights
(C) Number of electrons in the valency orbit
(D) The re-occurrence of similar outer electronic configuration

11. In the sixth period, the orbitals being filled are


(A) 5s, 5p, 5d (B) 6s, 6p, 6d, 6f
(C) 6s, 4f, 5d, 6p (D) 4s, 4f, 5d, 6p

Nothing is impossible, the word itself says ‘I’m possible’!


16 CHEMICAL PERIODICITY

BASED ON ATOMIC RADII


12. In the periodic table going down in fluorine group
(A) Reactivity will increase (B) Electronegativity will increase
(C) Ionic radius will increase (D) Ionization potential will increase

13. Radius of the isoelectronic species


(A) Increases with the increase of nuclear charge
(B) Decreases with the increase of nuclear charge
(C) Is the same for all (D) First increases and then decreases

14. In the isoelectronic species the ionic radii (Å) of N3 , O2 and F  are respectively given by
(A) 1.36, 1.40, 1.71 (B) 1.36, 1.71, 1.40
(C) 1.71, 1.40, 1.36 (D) 1.71, 1.36, 1.40

15. A trend common to both groups I and VII elements in the periodic table as atomic number
increases is
(A) Oxidising power increases (B) Atomic radius increases
(C) Maximum valency increases (D) Reactivity with water increases

16. The trivalent ion having largest size in lanthanide series is


(A) Ti (B) Zr (C) Hf (D) La
17. Atomic radii of fluorine and neon in Angstrom units are respectively given by :
(A) 0.72, 1.60 (B) 1.60, 1.60 (C) 0.72, 0.72 (D) None of these

BASED ON IONIZATION POTENTIAL

18. Which of the following electrons should have the highest value of ionisation energy (for the same
value of the principal quantum number)
(A) s (B) p (C) d (D) f

19. Which of the following relation is correct with respect to first (I) and second (II) ionization
potentials of sodium and magnesium
(A) IMg  IINa (B) INa  IMg (C) IIMg  IINa (D) IINa  IIMg

20. Which of the following species has lowest ionization potential


(A) O (B) O2 (C) O2 (D) O2

21. The second ionisation energy of N and O in electron volt are respectively given by
(A) 29, 29 (B) 34, 34 (C) 29, 34 (D) 34, 29

22. In which of the following pairs is there a change in the periodic trend for the ionisation energy -
(A) Fe—Ni (B) C—N (C) Be—B (D) O—F

23. The decreasing order of the first ionisation energies of the following elements is -
(A) Li  Be  B  C (B) C  B  Be  Li
(C) C  Be  B  Li (D) Be  B  Li  C

CatalyseR Eduventures (India) Pvt. Ltd.


CHEMICAL PERIODICITY 17

BASED ON ELECTRON AFFINITY


24. The electron affinity values for the halogens show the following trend
(A) F  Cl  Br  I (B) F  Cl  Br  I (C) F  Cl  Br  I (D) F  Cl  Br  I

25. Which of the following has the least electron affinity in kJmol1
(A) Oxygen (B) Carbon (C) Nitrogen (D) Boron

26. The electron affinity values (in kJmol1 ) of three halogens X, Y and Z are respectively – 349,
– 333 and – 325. Then X, Y and Z are respectively
(A) F2 ,Cl2 and Br2 (B) Cl2 ,F2 and Br2 (C) Cl2 ,Br2 and F2 (D) Br2 ,Cl2 and F2

27. Which of following species has the negative electron gain enthalpy?
(A) F (B) O (C) O (D) Na 
28. Which of the following pairs show reverse properties on moving along a period from left to right
and from top to down in a group
(A) Nuclear charge and electron affinity (B) Ionisation energy and electron affinity
(C) Atomic radius and electron affinity (D) None of these

29. Which one of the following arrangements represents the correct order of electron gain enthalpy
(with negative sign) of the given atomic species.
(A) Cl  F  S  O (B) O  S  F  Cl (C) S  O  Cl  F (D) F  Cl  O  S

30. The electron affinities of N,O,S and Cl are -


(A) N  O  S  Cl (B) O  N  Cl  S
(C) O = Cl  N = S (D) O  S Cl  N

31. The electron affinity values of elements A, B, C and D are respectively 135,  60, 200 and
348 kJ mol 1. The outer electronic configuration of B is :
(A) 3s 2 3 p5 (B) 3s 2 3 p 4 (C) 3s 2 3 p3 (D) 3s 2 3 p 2

BASED ON ELECTRO NEGATIVITY


32. Which of the following is most electronegative
(A) Carbon (B) Silicon (C) Lead (D) Tin

33. Which of the following sets of atoms is arranged in order of increasing electronegativity
(A) S, Si, P (B) S, P, Si (C) Si, P, S (D) Si, S, P

34. Choose the correct statement


(A) Electronegativity increases down a group
(B) Electronegativity decreases down a group
(C) Electronegativity decreases from left to right along a period
(D) Electronegativity changes along a group but remains constant along a period

Nothing is impossible, the word itself says ‘I’m possible’!


18 CHEMICAL PERIODICITY

35. Which is the correct order of electronegativities


(A) F > N < O > C (B) F>N>O>C
(C) F < N < O < C (D) F>N>O<C

36. For the formation of covalent bond, the difference in the value of electronegativities should be
(A) Equal to or less than 1.9 (B) More than 1.9
(C) 1.9 or more (D) None

BASED ON VALENCY & OXIDATION NUMBER

37. In the ground state of cobalt atom (Z = 27) there are ....... unpaired electrons and thus the atom
is.......
(A) 2, diamagnetic (B) 2, paramagnetic (C) 3, diamagnetic (D) 3, paramagnetic

38. Variable valency in general, is exhibited by


(A) Transition elements (B) Gaseous elements
(C) Non-metals (D) s -block elements

39. An element of atomic weight 40 has 2, 8, 8, 2 as the electronic configuration. Which one of the
following statements regarding this element is not correct
(A) It belongs to II group of the periodic table
(B) It has 20 neutrons
(C) The formula of its oxide is MO 2
(D) It belongs to 4th period of the periodic table

40. Which is the best reducing agent


(A) F (B) Cl (C) Br  (D) I

41. The maximum covalency of an element with atomic number 7 is


(A) 2 (B) 4 (C) 5 (D) 3

42. Thalium shows different oxidation states because


(A) It is a transition element (B) Of inert pair effect
(C) Of its amphoteric character (D) Of its higher reactivity

BASED ON MISCELLANEOUS TOPICS

43. The ion which has the maximum value of hydration energy is -
(A) Li+ (B) Na+ (C) K+ (D) Cs

44. The acid strength is


(A) ClOH  BrOH  IOH (B) BrOH  ClOH  IOH
(C) IOH  BrOH  ClOH (D) ClOH  IOH  BrOH

45. Which of the following does not reflect the periodicity of elements?
(A) Bonding behaviour (B) Electronegativity
(C) Ionisation potential (D) neutron / proton ratio

CatalyseR Eduventures (India) Pvt. Ltd.


CHEMICAL PERIODICITY 19

46. An atom of an element has electronic configuration 2, 8, 1. Which of the following statement is
correct
(A) The element's valency is 7 (B) The element exists as a diatomic molecule
(C) The element is of non-metallic nature (D) The element forms a basic oxide

47. Correct order of polarising power is


(A) Cs  K   Mg2  Al3  (B) K   Cs  Mg2  Al3 
(C) Cs  K   Al3  Mg2 (D) K   Cs  Al3  Mg2

48. The polarising power of which one of the following is highest


(A) Small & highly +ve ion (B) Large & highly +ve ion
(C) Small & highly –ve ion (D) Large & highly –ve ion

49. The most basic among these hydroxides, is


(A) Be(OH)2 (B) Mg(OH)2 (C) Ca(OH)2 (D) Ba(OH)2

50. Which of the following halogen acids is least acidic


(A) HI (B) HCl (C) HF (D) HBr

















Nothing is impossible, the word itself says ‘I’m possible’!


20 CHEMICAL PERIODICITY

EXERCISE # 02 OBJECTIVE EXERCISE (LEVEL # 02)


BASED ON ATOMIC & IONIC RADII

1. The size of the following species increases in the order:


(A) Mg2  Na  F (B) F  Na  Mg2
(C) Mg  F  Na (D) Na  F  Mg2

2. Highest size will be of


(A) Br  (B) I (C) I (D) I

3. Element Hg has two oxidation states Hg1 & Hg2 the right order of radii of these ions.

(A) Hg1  Hg2 (B) Hg2  Hg1 (C) Hg1  Hg2 (D) Hg2  Hg1

4. The correct order of increasing atomic size of element N, F, Si & P.


(A) N < F < Si < P (B) F > N < P < Si
(C) F < N < P < Si (D) F < N < Si < P

5. The correct order of atomic / ionic size


(A) N  Li  B (B) Cl  Mg  Ca
(C) Ca  2  S 2  Cl (D) Na  Mg2  Cl

BASED ON IONIZATION ENERGY OR POTENTIAL

6. In which of the following electronic configuration ionisation energy will be maximum in


(A) Ne 3s2 3p1 (B) Ne 3s 2 3p2 (C) Ne 3s 2 3p3 (D)  Ar  3d10 4s2 4p3
7. The correct order of second ionisation potential of C, N, O and F is:
(A) C > N > O > F (B) O > N > F >C (C) O > F > N > C (D) F>O>N>C

8. Decreasing ionization potential for K, Ca & Ba is


(A) Ba> K > Ca (B) Ca > Ba > K (C) K > Ba > Ca (D) K > Ca > Ba

9. The ionization energy will be maximum for the process.


(A) Ba  Ba   (B) Be  Be   (C) Cs  Cs  (D) Li  Li

10. The correct order of second I.P.


(A) Na < Mg > Al < Si (B) Na > Mg < Al > Si
(C) Na > Mg > Al < Si (D) Na > Mg > Al > Si
11. Alkaline earth metals always form dipositive ions due to
(A) IE2  IE1  10 (B) IE2  IE1  17 (C) IE2  IE1  10 (D) None of these

12. Amongst the following, the incorrect statement is


(A) IE1  Al   IE1  Mg  (B) IE1 Na   IE1 Mg 

(C) IE 2  Mg   IE 2  Na  (D) IE3  Mg   IE3  Al

CatalyseR Eduventures (India) Pvt. Ltd.


CHEMICAL PERIODICITY 21

BASED ON ELECTRON AFFINITY OR ELECTRON GAIN ENTHALPY

13. The electron affinity of the members of oxygen family of the periodic table, follows the sequence
(A) O > S > Se (B) S > O < Se (C) O < S > Se (D) Se > O > S

14. The process requires absorption of energy is


(A) F  F (B) Cl  Cl (C) O   O 2 (D) H  H

15. Of the following elements, which possesses the highest electron affinity?
(A) As (B) O (C) S (D) Se

16. Which of the following statement is not true?


(A) F atom can hold additional electron more tightly than Cl atom
(B) Cl atom can hold additional electron more tightly than F atom
(C) The incoming electron encounters greater repulsion for F atom than for Cl atom
(D) It is easier to remove an electron from F  than Cl .

17. Increasing order of Electron affinity for following configuration.


(A) 1s2,2s2 2p3 (B) 1s2,2s2 2p4
(C) 1s2 ,2s2 2p6 3s2 3p4 (D) 1s2 ,2s2 2p6 ,3s2 3p3
(A) a<d<b<c (B) d<a<c<b (C) a<b<c<d (D) a<b<d<c

18. Highest electron affinity is shown by


(A) F (B) Cl (C) Li (D) Na 

BASED ON ELECTRONEGATIVITY
19. In the following which configuration of element has maximum electronegativity.
(A) 1s2,2s2 2p5 (B) 1s2,2s2 2p6
(C) 1s2,2s2 2p4 (D) 1s2 ,2s2 2p6 ,3s2 3p3

20. On the Pauling’s electronegativity scale, which element is next to F .


(A) Cl (B) O (C) Br (D) Ne

21. The increasing order of acidic nature of Li2 O,BeO,B2 O3


(A) Li2O  BeO  B2 O3 (B) Li2O  BeO  B2 O3
(C) Li2O  BeO  B2 O3 (D) Li2O  BeO  B2 O3

22. Bond distance C  F in  CF4  & Si  F in  SiF4  are respective 1.33A & 1.54A . C–Si bond is

1.87A . Calculation the covalent radius of F atom ignoring the electronegativity differences.
1.33  1.54  1.8 
(A) 0.64A (B) A
3
1.54
(C) 0.5A (D) Å
2

Nothing is impossible, the word itself says ‘I’m possible’!


22 CHEMICAL PERIODICITY

23. Two elements A & B are such that B. E. of A–A, B–B & A–B are respectively 81 Kcal / mole,
64 Kcal / mole, 76 Kcal / mole & if electronegativity of B is 2.4 then the electronegativity of A may
be approximately
(A) 2.81 (B) 1.8 (C) 1.99 (D) 3.0

24. The lowest electronegativity of the element from the following atomic number is.
(A) 37 (B) 55 (C) 9 (D) 35

25. Which one is not correct order of electronegativity?


(A) F > Cl > Br > I (B) Si > Al > Mg > Na
(C) Cl > S > P > Si (D) None of these

26. Calculate the bond length of C–X bond if C – C bond length is 1.54A and X–X bond length is

1.2A and electronegativities of C and X are 2.0 and 3.0 respectively.


(A) 2.74A (B) 1.37A (C) 1.46A (D) 1.28A

BASED ON MISCELLANEOUS TOPICS

27. Choose the s-block element from the following:


(A) 1s2 ,2s2 ,2p6 ,3s2 ,3p6 ,3d5 ,4s1 (B) 1s2 ,2s2 ,2p6 ,3s2 ,3p6 ,3d10 ,4s1
(C) 1s2 ,2s2 ,2p6 ,3s2 ,3p6 ,4s1 (D) all of the above

28. False statement for periodic classification of elements is


(A) The properties of the elements are periodic function of their atomic numbers.
(B) No. of non-metallic elements is less than the no. of metallic elements.
(C) First ionization energy of elements does not increase regularly with the increasing of atomic
number in a period.
(D) d-subshell is filled by final electron with increasing atomic number of inner transition
elements.

29. Pick out the isoelectronic species from the following:



I. CH3 II. H3O III. NH3 IV. CH3
(A) I and II (B) III and IV
(C) I and III (D) II, III and IV

30. If there were 10 periods in the periodic table then how many elements would this period can
maximum comprise of.
(A) 50 (B) 72 (C) 32 (D) 98

31. Among the following which species is/are paramagnetic


(i) Sr 2  (ii) Fe3  (iii) CO2  (iv) S2 
(v) Pb2+
(A) i, iv, v (B) i, ii, iii (C) ii, iii (D) iv, v

CatalyseR Eduventures (India) Pvt. Ltd.


CHEMICAL PERIODICITY 23

32. If each orbital can hold a maximum of three electrons, the number of elements in 9th period of
periodic table (long form) are
(A) 48 (B) 162 (C) 50 (D) 75

33. The Zeff for


3d electron of Cr
4s electron of Cr
3d electron of Cr 3 
3s electron of Cr 3  are in the order respectively
(A) 4.6, 2.95, 4.95, 8.05 (B) 4.95, 2.95, 4.6, 8.05
(C) 4.6, 2.95, 5.3, 12.75 (D) none of these

34. Which among the following factors is most important in making fluorine, the strongest oxidising
halogen:
(A) Bond dissociation energy (B) Ionisation Enthalpy
(C) Hydration enthalpy (D) Electron affinity

35. Which of the following element has highest metallic character .


Element IP
(A) P 17 eV
(B) Q 2 eV
(C) R 10 eV
(D) S 13 eV

36. The electronic configuration of an element is 1s2 2s2 2p6 3s2 3p4 . The atomic number and the
group number of the element ‘X’ which is just below the above element in the periodic table are
respectively.
(A) 24 & 6 (B) 24 & 15 (C) 34 & 16 (D) 34 & 8

37. The correct order of increasing hydration energy of following ion is


(A) Fe 2  Co 2  Fe 3 (B) Fe 2  Co 2  Fe 3
(C) Fe 2  Co 2  Fe 3 (D) Fe 2  Fe 3  Co 2



Nothing is impossible, the word itself says ‘I’m possible’!


24 CHEMICAL PERIODICITY

EXERCISE # 03 OBJECTIVE EXERCISE (LEVEL # 03)


COMPREHENSION TYPE

Comprehension # 01 (Q.1 & Q.2)

Question No. 1 and 2 are based on the following information.


Four elements P, Q, R & S have ground state electronic configuration as:
P  1s 2 2 s 2 2 p 6 3s 2 3 p3 Q  1s 2 2 s 2 2 p 6 3s 2 3 p1
R  1s 2 2s 2 2 p 6 3s 2 3 p 6 3d 10 4s 2 4 p3 S  1s 2 2s 2 2 p 6 3s 2 3 p 6 3d 10 4s 2 4 p1

1. Comment which of the following option represent the correct order of true (T) & false (F)
statement.
I. size of P < size of Q II. size of R < size of S
III. EA of P < EA of R IV. EA of Q > EA of S
(A) TTTT (B) TTFT (C) TFFT (D) TTFF

2. Order of IE1 values among the following is


(A) P>R>Q>S (B) P<R<S<Q (C) R>S>P>Q (D) P>S>R>Q

Comprehension # 02 (Q.3 to Q.5)

Nature of bond can be predicted on the basis of electronegativity of bonded atoms, greater
difference in electronegativity (X), more will be the polarity of bond, and polar bond are easily
broken in polar solvent like water. For hydroxy acids X O  X A difference predict the nature of
oxide formed by the element A.
X O  X A  X O  X H then A–O–H show basic nature (NaOH)
X O  X A  X O  X H then A–O–H show acidic nature (H–O–Cl)
With the help of EN values  EN A  1.8, EN B  2.6, EN C  1.6, EN D  2.8 answer the following
questions for the compounds HAO, HBO, HCO, HDO.

3. Compounds whose aqueous solution is acidic and order of their acidic strength
(A) AOH, COH ; AOH < COH (B) HDO, HBO ; HDO > HBO
(C) AOH, COH ; AOH > COH (D) HDO, HBO ; HDO < HBO

4. Compounds whose aqueous solution is basic and order of their basic strength
(A) AOH, COH ; AOH < COH (B) HDO, HBO ; HDO > HBO
(C) AOH, COH ; AOH > COH (D) HDO, HBO ; HDO < HBO

5. Percentage ionic character of compound AB is


(A) 42.42% (B) 24.24% (C) 15.04% (D) None of these

CatalyseR Eduventures (India) Pvt. Ltd.


CHEMICAL PERIODICITY 25

Comprehension # 03 (Q.6 to Q.8)

Pauling gave method to calculate univalent ion radii by assuming that


 
(i) In ionic crystal (let M X ) cations and anions are is contact of each other and sum of their radii
is equal to interionic distance, i.e.
d( M  — X ¯)  rM   rX 
(ii) The radius of an ion having noble gas configuration is inversely proportional to the effective
C C
nuclear charge felt at the periphery of the ion, i.e. r  and r 
M 
Z eff . M  X 
Z eff . X 
   
Here C is constant of proportionality whose value depends on electronic configuration of ion.
Thus,
C C
d( M   X ¯)   pm
Z eff .( M  ) Z eff . ( X ¯)

Z eff . is the effective nuclear charge whose value can be calculated by the formula :
Z eff .  Z   . Here  is shielding constant and for neon, the value of  when calculate by
Slater’s rule, is found to be 4.5.

6. The value of constant C for NaF crystals is [given that interionic distance of NaF = 231 pm] :
(A) 231 (B) 115.5 (C) 614.5 (D) 307.25


7. The value of univalent radii for F as calculated by Pauling method is (given that interionic
distance of NaF = 231 pm) :
(A) 94.5 pm (B) 136.5 pm (C) 111.68 pm (D) 115.5 pm

8. The value of ‘C’ for Na  , Mg 2 and Al 3 will be in the order :


(A) Al 3  Mg 2  Na  (B) Al 3  Mg 2  Na 
(C) Al 3  Mg 2  Na  (D) Can’t be compared.

Comprehension # 04 (Q.9 to Q.13)


All the simple salt dissolve in water, producing ions and consequently the solution conduct
electricity. In this process water molecule surround both the cations and anions & release energy.
This process is called hydration & energy released is called hydration energy & it depends on
size of gaseous ions.
Answer the following questions with respect to given cations.
Na  , Mg 2 , Al 3
9. Order of extent of hydration
(A) Na   Mg 2  Al 3 (B) Na   Mg 2  Al 3
(C) Al 3  Mg 2  Na  (D) Al 3  Mg 2  Na 

Nothing is impossible, the word itself says ‘I’m possible’!


26 CHEMICAL PERIODICITY

10. Order of hydration energy


(A) Na   Mg 2  Al 3 (B) Na   Mg 2  Al 3
(C) Al 3  Mg 2  Na  (D) Al 3  Mg 2  Na 

11. Order of size of hydrated ion.


(A) Na   Mg 2  Al 3 (B) Na   Mg 2  Al 3
(C) Al 3  Mg 2  Na  (D) Al 3  Mg 2  Na 

12. Order of ionic mobility


(A) Na   Mg 2  Al 3 (B) Na   Mg 2  Al 3
(C) Al 3  Mg 2  Na  (D) Al 3  Mg 2  Na 

13. Order of size of gaseous ions.


(A) Na   Mg 2  Al 3 (B) Na   Mg 2  Al 3
(C) Al 3  Mg 2  Na  (D) Al 3  Mg 2  Na 

Comprehension # 05 (Q.14 to Q.18)

According to Hess’s law enthalpy change for a process in independent on number of steps
involved, whether a process is carried out in one or several steps. Net enthalpy change remain
same.
For example a process carried out from A to D via intermediate step B and C then enthalpy
change for step A to D is equal to some of enthalpy change for A to B, B to C and C to D.
HA–B HB–C
A B C
HC–D
HA–D
D
H A D  H A B  H B C  H C  D
With the help of informations given below answer the following questions

CatalyseR Eduventures (India) Pvt. Ltd.


CHEMICAL PERIODICITY 27

14. H L. E . of NaAlCl4 will be _____________.


(A) 230 (B) 240 (C) 250 (D) 210

15. H H. E . of Al 3  g  ion will be _____________.


(A) 20 (B) 40 (C) 45 (D) 60
16. H f of NaCl  s  will be _____________.
(A) 100 (B) 90 (C) 80 (D) 110
17. BECl2  H eg Cl  g  will be _____________.
(A) 100 (B) 80 (C) 90 (D) 110
3
18. H eg1 of  Al  g   will be _____________.

(A) 30 (B) 40 (C) 20 (D) 50

Comprehension # 06 (Q.19 to Q.21)

The electronic configuration of the elements show a periodic variation of increasing atomic
number. Consequently there are also periodic variations in physical and chemical behaviour of
the elements. Before examining the variation in properties, the concept of effective nuclear
charge should also be understood as it influences the atomic size and ionization energy. The
effective nuclear charge effect is the charge felt by the valence electron Zeff = Z   , where Z is
actual nuclear change and  is screening constant. Now variation in different type of atomic radii
i.e. metallic radius covalent radius and Vander Wall radius can be studied keeping Zeff in
consideration. Zeff also plays an important role in variation of ionization energy across the period
and group. A large Zeff means a more tightly held outer electron and hence greater first ionization
energy.

19. The increasing order of effective nuclear charge in Na, Al, Mg and Si atoms
(A) Na < Mg < Si < Al (B) Na < Mg < Al < Si
(C) Mg < Na < Al < Si (D) Na = Mg = Al = Si

20. Which of the following option is correct? [State T for true and F for false]
(i) On moving from left to right in a period screening constant value is increase
(ii) In a s, p, d, f-orbitals the order of screening effect is s > p > d > f in a given shell
(iii) In any period from left to right effective nuclear charge increases so electronegativity is
also increases
(iv) When  increases, Zeff also increase
(A) TTTF (B) FFFT (C) FFTT (D) TTFF

21. Zeff (I)............. along the period and (II).............. along the group.
(A) (I) Increases, (II) Increases (B) (I) decreases, (II) Increases
(C) (I) decreases, (II) decreases (D) (I) Increases, (II) Almost same

Nothing is impossible, the word itself says ‘I’m possible’!


28 CHEMICAL PERIODICITY

MORE THAN ONE MAY BE CORRECT

22. Which of the following is correct order of EA.


(A) N<C<O<F (B) F > Cl > Br > I
(C) Cl > F > Br > I (D) C<N<O<F

23. Select the correct statement(s).


(A) The value of electron gain ethalpy of an element can be -ve or +ve.
(B) In the periodic table, metallic character of the elements increases down the group and
decreases across the period
(C) The Cl  & S 2 are isoelectronic species but first one is not smaller in size than the second
(D) Ionization enthalpy of an atom is equal to electron gain enthalpy of cation

24. In which of the following are the orders of electron affinity of the elements or ions shown correctly
(A) S  O (B) N  P (C) O  S  (D) O  S

25. Which of the following parameters can’t be estimated by using Born-Haber cycle (for the
formation of the ionic compound)?
(A) Hydration energy (B) Electron gain enthalpy
(C) Lattice energy (D) Electronegativity

26. Which of the following are correct


(A) IE2  Mg   IE2  Na  (B) EA  N   EA  P 

(C) Atomic size Mg 2  Atomic size  Li   (D) IP of Na < Mg < Al

27. If Aufbau’s principle and Hund’s rule were not followed.


(A) K would have been d–block element & paramagnetic.
(B) Cu would have been s–block element.
(C) Cr would have been diamagnetic
(D) Fe3 ion would have 5 unpaired electrons.

28. In halogen, which of the following properties increase from iodine to fluroine
(A) Ionisation energy (B) Electronegativity
(C) Bond length (D) Electron affinity

29. Which of the following pair have nearly the same atomic radii
(A) Al and Ga (B) Fe and Ni
(C) Zr and Hf (D) Pt & Pd

30. In which of the following sets of elements 1st element is more metallic then second.
(A) Ba, Ca (B) Sb, Sn
(C) Ge, S (D) Na, F

CatalyseR Eduventures (India) Pvt. Ltd.


CHEMICAL PERIODICITY 29

31. Amongst the following statements, which is / are correct?


(A) Electronegtaivity of sulphur is greater than that of oxygen.
(B) Electron affinity of oxygen is smaller than that of sulphur.
(C) Electron affinity of fluorine is most negative
(D) Electron affinity of chlorine is most negative
 
32. The ionic compound A B is formed when the
(A) electron affinity of B is high (B) ionization energy of A is low
(C) lattice energy of AB is high (D) lattice energy of AB is low

33. Which of the following is/are correct?


(A) For A  g   e  
 A  g   H should be negative
(B) For A   g   e  
 A2   g   H should be negative
(C) For A   g   e  
 A2   g   H should be positive
(D) For Ne  g   e  
 Ne   g   H should be zero

34. Which of the following statements is/are correct ?


(A) Tl 3 salts are oxidising agents (B) Ga  salts are reducing agents
(C) Pb 4 salts are better oxidising agents (D) As5 salts are oxidising agents
35. Which one of the following statement (s) is (are) correct?
(A) The electronic configuration of Cr is [Ar] 3d5 4s1.(Atomic No. of Cr = 24)
(B) The magnetic quantum number may have a negative value
(C) In silver atom, 23 electrons have a spin of one type and 24 of the opposite type. ( Atomic
No. of Ag = 47)
(D) The oxidation state of nitrogen in HN3 is –3.

36. Which of the following statements are correct?


(A) Helium has the highest first ionization enthalpy in the periodic table.
(B) Chlorine has less negative electron gain enthalpy than fluorine.
(C) Mercury and bromine are liquids at room temperature.
(D) In any period, atomic radius of alkali metal is highest.

MATCH THE COLUMN

37. Match the electronic configuration (list - I) with the ionization energy (List - II)
List - I List - II
(Electronic Configuration) (Ionization Energy kJ/mol)
(A) n s2 (P) 2100
(B) n s2 n p1 (Q) 1400
(C) n s2 n p3 (R) 800
(D) n s2 n p6 (S) 900

Nothing is impossible, the word itself says ‘I’m possible’!


30 CHEMICAL PERIODICITY

38. If electrons are filled in the sub shells of an atom in the following order 1s, 2s, 2p, 3s, 3p, 3d, 4s,
4p, 4d, 4f......... then match the following element in List I with block in List II.
List-I List-II
(A) K(19) (P) s-Block
(B) Fe(26) (Q) p-Block
(C) Ga(31) (R) d-Block
(D) Sn(50) (S) f-block

39. Match the characteristics mentioned in List II with the process in List I.
List I List II
(A) O  g   e  O   g  (P) Positve electron affinity
(B) O   g   e  O 2  g  (Q) Negative electron affinity

(C) Na   g   Na  g   e  (R) Exothermic


(D) Mg  g   e   Mg  g  (S) Endothermic

40. Match the column :


Column I Column II
(A) Highest density (P) Lithium
(B) Metallic character (Q) Osmium
(C) Lightest Metal (R) Mercury
(D) Liquid at room temperature (S) Bromine

41. Match the column :


Column I Column II
(A) Cl (P) Non Metallic or Electronegative element
(B) F (Q) Highest electron affinity
(C) Cu (R) Variable covalency existance
(D) He (S) Highest ionisation energy.

42. Match the values of ionization energy and electron gain enthalpy listed in column I with
characteristics(s) of elements listed in column II.
Column I Column II
1 H 1 , 1 H 2 ,  eg H  in KJ mol 1

(A) 2372 5251  48 (P) Element which acts as a strong reducing agent
(B) 419 3051  48 (Q) Element which exists as a monoatomic molecule
(C) 1661 3374  333 (R) Least reactive non-metal
(D) 1008 1846 295 (S) Element which acts as a strong oxidizing agent
(T) Element which oxide is a stronger basic in nature

CatalyseR Eduventures (India) Pvt. Ltd.


CHEMICAL PERIODICITY 31

43. Match the increasing orders given in column I with the property(ies) given in column II.
Column I Column II
(A) Na  F   O 2  N 3 (P) Electronegativity
(B) Li   Na  K   Rb  Cs  (Q) Nuclear charge
(C) O  S  F  Cl (R) Size
(D) Cl   K   Ca 2  Sc3 (S) Electron affinity
(T) Ionisation energy

INTEGER ANSWER TYPE QUESTION

44. Give the atomic number of the inert gas atom in which the total number of d-electrons is equal to
the difference in the number of total p  and s  electrons.

45. The Lanthanides and Actinides are placed in group number:


18
46. A metal has electronic configuration  Ar  3d7 4s2 . On the basis of this electronic configuration
find out the group number.

47. Amongst the following, the total number of orders which are correct with respect to the property
indicated against each is :
(i) Mg  Al  Si  P Covalent radius
(ii) Na  O2  F  N3  Ionic size
(iii) Al3  Mg2  Li  K  Ionic size

(iv) C  Si  P  N Electron affinity value


(v) NCOF Electron affinity value
(vi) F  Cl  Br  I Electron affinity value
(vii) Si  Mg  Al  Na First ionization energy
(viii) OFNC Second ionization energy
(ix) N  P  Sb  As Third ionization energy

48. An ion having a 4+ charge and a mass of 51.99 amu has two electrons with n = 1, eight electrons
with n = 2, and ten electrons with n = 3. Give the total number of protons present in the nucleus of
the atom of metal.

49. Total number of elements which are belong to same period (III).
Li, Na, Mg, F, Ne, Sc, P, Ar

50. The value of n (i.e. principal quantum number) for the valence shell of palladium is (atomic
number = 46) :


Nothing is impossible, the word itself says ‘I’m possible’!


32 CHEMICAL PERIODICITY

EXERCISE # 04 SUBJECTIVE EXERCISE (LEVEL # 01)


1. Use the following system of naming elements in which first alphabets of the digits are written
collectively,
0 1 2 3 4 5 6 7 8 9
nil uni bi tri quad pent hex sept oct enn
to write three-letter symbols for the elements with atomic number 101 to 109.
[Example : 101 is Unu....]

2. Mg2 ,O2 ,Na ,F ,N3 (Arrange in decreasing order of ionic size)

3. Why Ca 2  has a smaller ionic radius than K  .


4. Arrange in decreasing order of atomic size : Na, Cs, Mg, Si, Cl.
5. Why the first ionisation energy of carbon atom is greater than that of boron atom whereas, the
reverse is true for the second ionisation energy.
6. The IE do not follow a regular trend in II & III periods with increasing atomic number. Why?
7. Explain why a few elements such as Be (+0.6), N(+0.3) & He(+0.6) have positive electron gain
enthalpies while majority of elements do have negative values.
8. Which bond in each pair is more polar
(A) P – Cl or P – Br (B) S – Cl or S – O (C) N – O or N – F
9. State giving reasons which one have higher value :
(A) IE1 of F or Cl (B) E A of O or O 
(C) ionic radius of K  or Cl
10. From among the elements, choose the following: Cl, Br, F, Al, C, Li, Cs & Xe.
(i) The element with highest electron affinity.
(ii) The element with lowest ionisation potential.
(iii) The element whose oxide is amphoteric.
(iv) The element which has smallest radii.
(v) The element whose atom has 8 electrons in the outermost shell.

11. In the ionic compound KF, the K  and F  ions are found to have practically radii, about 1.34A
each. What do you predict about the relative covalent radii of K and F?
12. Which oxide is more basic, MgO or BaO? Why?
13. The basic nature of hydroxides of group 13 (III-A) decreases progressively down the group.
Comment.
14. Based on location in P.T., which of the following would you expect to be acidic & which basic.
(A) CsOH (B) IOH (C) Sr  OH2 (D) Se  OH2 (E) FrOH (F) BrOH

15. Compare the following giving reasons


Acidic nature of oxides: CaO,CO,CO2 ,N2 O5 ,SO3


CatalyseR Eduventures (India) Pvt. Ltd.


CHEMICAL PERIODICITY 33

EXERCISE # 05 SUBJECTIVE EXERCISE (LEVEL # 02)



1. If internuclear distance between Cl atoms in Cl2 is 10A & between H atoms in H2 is 2A , then
calculate internuclear distance between H & Cl (Electronegativity of H = 2.1 & Cl = 3.0).

2. The As-Cl bond distance in AsCl3 is 2.20A . Estimate the SBCR (single bond covalent radius) of

As. (Assume EN of both to be same and radius of Cl  0.99A )

3. The Pt-Cl distance has been found to be 2.32A in several crystalline compounds. If this value
applies to both of the compounds shown in figure. What is Cl - Cl distance in (A) and (B)

(A) (B)
4. The IE values of Al g  Al  e is 577.5kJmol1 and H for Alg  Al3   3e is 5140kJmol1. If

second and third IE values are in the ratio 2 : 3. Calculate IE2 and IE3 .

5. For the gaseous reaction,


K  F  K F , H was calculated to be 19 kcal/mol under conditions where the cations and
anions were prevented by electrostatic separation from combining with each other. The ionisation
potential of K is 4.3 eV/atom. What is the electron affinity of F?
6. The ionisation potentials of atoms A and B are 400 and 300 kcal mol1 respectively. The electron
affinities of these atoms are 80.0 and 85.0 k cal mol1 respectively. Prove that which of the atoms
has higher electronegativity.

7. Calculate E.N. of chlorine atom on Pauling scale if I.E. of Cl is 4eV & of E.A. of Cl is +13.0eV.

8. Calculate the electronegativity of fluorine from the following data :


EHH  104.2kcalmol1 EF F  36.2kcalmol1
EHF  134.6kcalmol1 xH  2.1
9. Calculate the E.N. of Cl from the bond energy of ClF (61 Kcal/mol). Given that bond energies of
F2 and Cl2 are 38 and 58 Kcal/mol respectively.

10. How many chlorine atoms will be ionised Cl  Cl  e  1 by the energy released from the
process Cl  e 1  Cl for 6.02  1023 atoms
(I.P. for Cl  1250kJmol1 and E.A.  350kJmole1 )

11. A mixture contains F and Cl atoms. The removal of an electron from each atom of the sample
requires 284 kJ while the addition of an electron to each atom of the mixture releases 68.8 kJ.
Determine the % composition of the mixture.
(IE) per atom (EA) per atom
22
F 27.91 10 kJ 5.53  1022 kJ
Cl 20.77  1022 kJ 5.78  1022 kJ

Nothing is impossible, the word itself says ‘I’m possible’!


34 CHEMICAL PERIODICITY

12. Calculate the lattice energy of NaCl crystal from the following data by the use of Born-Haber
cycle. Sublimation energy of Na = 26 kcal/g. atom, dissociation energy of Cl2  54kcal / mole,
ionisation energy for Na(g) = 117 kcal/ mol, electron affinity for Cl(g) = 84 kcal/g atom, heat of
formation of NaCl = – 99 kcal/mole.
13. Calculate the electron affinity of iodine with the help of the following data (given in Kcal/mole).
1
 Hfor NaI  68.8,  Hsub Na  25.9,  Hsub  Hdiss I  25.5,
2 2

IP Na  118.4, UNaI  165.4


14. From the following information
A   g   A 2  g   3e  H1  1400kJ
2 
A  g   A  aq   2e H2  700kJ

HEG  A  g   350kJ / mol
IE1  IE 2  for A  g  950 kJ / mol
Find
(A) IE1 of A (B) IE2 of A
(C) HEG of A (D) HHE of A 2   g 

15. For ionic compound A 2B22 & C22D2


(A) A 4  s   B 2  g   AB2  s  Hf  500
Find the lattice energy of AB2
(B) Find Hf for according D3  g   C  s   C 2D  s 
Given on sublimation of above metal  A 4  it dissociates into individual atom.
Given : Sublimation energy of A 4 is 1600 ; Sublimation energy of C is 100
Dissociation energy of B 2 is 200 ; Dissociation energy of D3 is 90
A  g   A   g  ; H  50
 2
A  g  A  g  ; H  150

B  g   B  g  ; H  260
B   g   B 2  g ; H  250
D  g    D g  ; H  50
D   g   D2   g  ; H  100
C   g   C g ; H  150
C 2D  s   2C   g   D2   g  ; H  1000
All the values are in kJ/mol
16. Using the concept of Zeff (from Slater's rule). Explain the following:
"In obtaining the electronic configuration of V  from that of V, an e  is removed from
4s and not from 3d."







CatalyseR Eduventures (India) Pvt. Ltd.


CHEMICAL PERIODICITY 35


EXERCISE # 06 KVPY, OLYMPIADS CORNER


1. The isoelectronic pair is : [KVPY 2014]
(A) CO, N 2 (B) O2 , NO (C) C2 , HF (D) F2 , HCl

2. The first ionization enthalpies for three elements are 1314, 1680 and 2080 kJ mol 1 ,
respectively. The correct sequences of the elements is - [KVPY 2014]
(A) O, F and Ne (B) F , O and Ne (C) Ne, F and O (D) F , Ne and O

3. Mendeleev’s periodic law states that the properties of elements are a periodic function of their
[KVPY 2015]
(A) Reactivity of elements (B) Atomic size
(C) Atomic mass (D) Electronic configuration

4. Of the following, the ion with the largest size is - [NSEC 2014]
(A) O2  (B) Na (C) F (D) Al 3



Nothing is impossible, the word itself says ‘I’m possible’!


36 CHEMICAL PERIODICITY

EXERCISE # 07 JEE (MAIN) CORNER


1. The increasing order of the first ionization enthalpies of the elements B, P, S and F (lowest first) is:
[2006]
(A) F<S<P<B (B) P<S<B<F
(C) B<P<S<F (D) B<S<P<F

2. Which one of the following sets of ions represents a collection of isoelectronic species? [2006]
(A) K+, Cl–, Ca2+, Sc3+ (B) Ba2+, Sr2+, K+, S2–
(C) N3–, O2–, F–, S2– (D) Li+, Na+, Mg2+, Ca2+

3. Which one of the following constitutes a group of the isoelectronic species? [2008]
(A) NO+, C22–, CN–, N2 (B) CN–, N2, O22–, C22–

(C) N2, O2–, NO+, CO (D) C22–, O2–, CO, NO

4. The set representing the correct order of ionic radius is : [2009]


(A) Na+ > Li+ > Mg2+ > Be2+ (B) Li+ > Na+ > Mg2+ > Be2+
(C) Mg2+ > Be2+ > Li+ > Na+ (D) Li+ > Be2+ > Na+ > Mg2+

5. In which of the following arrangements, the sequence is not strictly according to the property
written against it ? [2009]
(A) HF < HCl < HBr < HI : increasing acid strength
(B) NH3 < PH3 < AsH3 < SbH3 : increasing basic strength
(C) B < C < O < N : increasing first ionization enthalpy
(D) CO2 < SiO2 < SnO2 < PbO2 : increasing oxidising power

6. The correct sequence which shows decreasing order of the ionic radii of the elements is : [2010]
(A) Al3+ > Mg2+ + Na+ > F– > O2– (B) Na+ > Mg2+ > Al3+ > O2– > F–
(C) Na+ > F– > Mg2+ > O2– > Al3+ (D) O2– > F– > Na+ > Mg2+ > Al3+

7. Which of the following represents the correct order of increasing first ionization enthalpy for Ca,
Ba, S, Se and Ae? [2013]
(A) Ca < S < Ba < Se < Ar (B) S < Se < Ca < Ba < Ar
(C) Ba < Ca < Se < S < Ar (D) Ca < Ba < S < Se < Ar

8. The first ionisation potential of Na is 5.1 eV. The value of electron gain enthalpy of Na+ will be:
[2013]
(A) 2.55 eV (B) 5.1eV (C) 10.2 eV (D) 2.55 eV

9. Among the following oxoacids, the correct decreasing order of acid strength is : [2014]
(A) HClO2  HClO 4  HClO3  HOCl (B) HOCl  HClO2  HClO3  HClO4
(C) HClO 4  HOCl  HClO2  HClO3 (D) HClO4  HClO3  HClO2  HOCl

CatalyseR Eduventures (India) Pvt. Ltd.


CHEMICAL PERIODICITY 37

10.

The ionic radii  in A  of N 3 , O 2 and F  are respectively - [2015]
(A) 1.36, 1.71 and 1.40 (B) 1.71, 1.40 and 1.36
(C) 1.71, 1.36 and 1.40 (D) 1.36, 1.40 and 1.71

11. Which of the following atoms has the highest first ionization energy? [2016]
(A) Na (B) K
(C) Sc (D) Rb

12. The group having isoelectronic species is : [2017]


   2  2
(A) O , F , Na, Mg (B) O , F , Na, Mg
(C) O  , F  , Na  , Mg 2 (D) O 2 , F  , Na  , Mg 2



Nothing is impossible, the word itself says ‘I’m possible’!


38 CHEMICAL PERIODICITY

EXERCISE # 08 JEE (ADVANCED) CORNER


4 4
1. STATEMEBT – 1 : Pb compounds are stronger oxidizing agent than Sn compounds.
[2008]
and
STATEMEBT – 2 : The higher oxidation states for the group 14 elements are more stable for the
heavier members of the group due to ‘inert pair effect’.
(A) STATEMENT-1 is True, STATEMENT-2 is True; STATEMENT-2 is correct explanation for
STATEMENT-1
(B) STATEMENT-1 is True, STATEMENT-2 is True; STATEMENT-2 is NOT a correct
explanation for STATEMENT-1
(C) STATEMENT-1 is True, STATEMENT-2 is False
(D) STATEMENT-1 is False, STATEMENT-2 is True

2. Among the following, the number of elements showing only one non-zero oxidation state is
O, Cl , F , N , P, Sn, Tl , Na, Ti [2010]

3. The increasing order of atomic radii of the following Group 13 elements is [2016]
(A) Al  Ga  In  Tl (B) Ga  Al  In  Tl
(C) Al  In  Ga  Tl (D) Al  Ga  Tl  In
4. The option(s) with only amphoteric oxides is (are): [2017]
(A) Cr2O3 , BeO, SnO, SnO2 (B) Cr2O3 , CrO, SnO, PbO
(C) ZnO, Al2O3 , PbO, PbO2 (D) NO, B2O3 , PbO, SnO2



CatalyseR Eduventures (India) Pvt. Ltd.


CHEMICAL PERIODICITY 39

ANSWER KEYS
EXERCISE # 01 OBJECTIVE EXERCISE (LEVEL # 01)

1 2 3 4 5 6 7 8 9 10
A A B A A C D B C D
11 12 13 14 15 16 17 18 19 20
C C B C B D A A D D
21 22 23 24 25 26 27 28 29 30
C C C A C B B C B A
31 32 33 34 35 36 37 38 39 40
C A C B A A D A C D
41 42 43 44 45 46 47 48 49 50
B B A A D D A A D C

EXERCISE # 02 OBJECTIVE EXERCISE (LEVEL # 02)


1 2 3 4 5 6 7 8 9 10
A C A D B C C B B B
11 12 13 14 15 16 17 18 19 20
C C C C C A A C A B
21 22 23 24 25 26 27 28 29 30
B C A B D D C D D B
31 32 33 34 35 36 37
C D C C B C A

EXERCISE # 03 OBJECTIVE EXERCISE (LEVEL # 03)


1 2 3 4 5 6 7 8 9 10
B A B A C C B D C C
11 12 13 14 15 16 17 18 19 20
C B B A B B D C B A
21 22 23 24 25 26 27 28 29 30
D AC ABD AD AD AB ABC AB ABCD ACD
31 32 33 34 35 36 37
BD ABC AC ABC ABC ACD (A) – (S), (B) – (R), (C) – (Q), (D) – (P)
38 39
(A) – (R), (B) – (R), (C) – (Q), (D) – (S) (A) – (QR), (B) – (PS), (C) – (S), (D) – (QR)

Nothing is impossible, the word itself says ‘I’m possible’!


40 CHEMICAL PERIODICITY

40 41
(A) – (PQR), (B) – (P), (C) – (R), (D) – (S)
(A) – (Q), (B) – (PQR), (C) – (P), (D) – (RS)

42 43
(A) – (Q), (B) – (PT), (C) – (S), (D) – (R) (A) – (R), (B) – (QR), (C) – (S), (D) – (PQST)
44 45 46 47 48 49 50
36 3 9 6 (ii), (vi), (ix) are incorrect. 24 4 4

EXERCISE # 04 SUBJECTIVE EXERCISE (LEVEL # 01)


Q.1 101 102 103 104 105 106 107 108 109
Unu Unb Unt Unq Unp Unh Uns Uno Une
Q.2 N3– > O2– > F– > Na+ > Mg2+
Q.3 Isolelectronic Ca+2(Value of Zeff is higher)

Q.4 Cs > Na > Mg > Si > Cl Q.5 Zeff & half filled config.
Q.6 half filled & fully filled orbitals Q.7 half filled and fully filled orbitals

Q.8 (a) P–Cl (b) S–O, (C) N–F Q.9 (a) F (b) O (c) Cl–
Q.10 (i) Cl (ii) Cs (iii) Al (iv) F (v) Xe Q.11 rk > 1.34Å > rF

Q.12 BaO Q.13 False


Q.14 (a) basic (b) acidic (c) basic (d) acidic (e) basic (f) acidic
Q.15 CaO < CO < CO2 < N2O5 < SO3

EXERCISE # 05 SUBJECTIVE EXERCISE (LEVEL # 02)


Q.1 5.919 Å
Q.2 1.21 Å Q.3 4.64 Å ; b = 3.28 Å
Q.4 IE2 = 1825 kJ/mole, IE3 = 2737.5 kJ/mol Q.5 3.476 eV

Q.6 EN1 > EN2 Q.7 3.03 (Pauling)

Q.8 3.8752 Q.9 3.2


Q.10 1.686 × 1023 atom Q.11 F = 37.81%, Cl = 62.19%
Q.12 –185 kcal/mole Q.13 +73.2 kcal/mole
Q.14 (a) IE1 of A = 350 kJ/mol, (b) IE2 = 600 kJ/mol,
(c) EGH of A = –450 kJ/mol, (d) EH of (A2+) = –250 kJ/mol
Q.15 – 780, – 420, AB > CD Q.16 Zeff = 4.3

CatalyseR Eduventures (India) Pvt. Ltd.


CHEMICAL PERIODICITY 41

EXERCISE # 06 KVPY, OLYMPIADS CORNER

1 2 3 4
A OR D A C A

EXERCISE # 07 JEE (MAIN) CORNER

1 2 3 4 5 6 7 8 9 10
D A A A A D C B D
11 12
C D

EXERCISE # 08 JEE (ADVANCED) CORNER

1 2 3 4
C 2 B AC

Nothing is impossible, the word itself says ‘I’m possible’!


ELECTROSTATICS 1

CHEMICAL BONDING
INDEX

 CONCEPTS IN BRIEF (CHEMICAL BONDING) 43 - 54


 SOLVED EXAMPLES 55 - 61
 EXERCISE # 01 OBJECTIVE EXERCISE (LEVEL # 01) 62 - 66
 EXERCISE # 02 OBJECTIVE EXERCISE (LEVEL # 02) 67 - 72
 EXERCISE # 03 OBJECTIVE EXERCISE (LEVEL # 03) 73 - 83
 EXERCISE # 04 SUBJECTIVE EXERCISE (LEVEL # 01) 84 - 85
 EXERCISE # 05 SUBJECTIVE EXERCISE (LEVEL # 02) 86 - 87
 EXERCISE # 06 KVPY, OLYMPIADS QUESTIONS 88 - 89
 EXERCISE # 07 JEE (MAIN) CORNER 90 - 92
 EXERCISE # 08 JEE (ADVANCED) CORNER 93 – 96
 ANSWER KEYS 97 – 99

WEIGHTAGE OF ‘CHEMICAL BONDING’ IN JEE (MAIN & ADVANCED) in Last Three Years

JEE (MAIN) Formely known as AIEEE

MARK /
YEAR No. Of Qs.
CHEMISTRY TOTAL MARKS
2015 0 0/120
2016 1 4/120
2017 1 4/120

JEE (ADVANCED)

MARK /
YEAR No. Of Qs.
CHEMISTRY TOTAL MARKS
2015 1 4/168
2016 0 0/124
2017 4 14/122

Nothing is impossible, the word itself says ‘I’m possible’!


CHEMICAL BONDING 43

CHEMICAL BONDING
CHEMICAL BOND :

A force that acts between two or more than two atoms to hold them together as a stable
molecule is known as a chemical bond.

REASONS FOR BOND FORMATION:

Lowerisation of energy due to attractions.


Attainment of Octet ns 2 np 6  , assumed to be most stable.

LEWIS SYMBOLS :

In the formation of a molecule, only the outer shell electrons take part in chemical combination
and they are known as valence electrons.
For example, the Lewis symbols for the elements of second period are as under:

TYPES OF BONDS : Ionic, covalent, co-ordinate

IONIC BOND (ELECTROVALENT BOND)


(i) It involves complete transfer of one or more electrons from the valence shell to the other atom.
Ionic bond is always form between electro positive and electro negative elements.

complete octet of e 

or Mg + 2 (F – ) 2

(ii) The capacity of an element in terms of number of loss of e– or gain of electron to form ionic bond
is termed electrovalency

Nothing is impossible, the word itself says ‘I’m possible’!


44 CHEMICAL BONDING

FAVOURABLE CONDITION :

(i) Metal should have low .E value.


(ii) Non metal should have more negative value of electron gain enthalpy.
(iii) Energy released because of the combination of cation and anion should be high (Lattice
enthalpy).

LATTICE ENTHALPY :

The Lattice Enthalpy of an ionic solid is defined as the energy required to completely separate
one mole of a solid ionic compound into gaseous constituent ions.
Na+(g) + Cl–(g)  NaCl(s) H = Hlattice  –ve (always)

FACTORS AFFECTING L.E.


1
(i) Lattice energy (L.E.)  r = r+ + r– = interionic distance
r
(ii) L.E.  Z+, Z–
Z+  charge on cation in terms electronic charge
Z–  charge on anion in terms electronic charge

(iii) L.E.  cordination number (C.N.)

CALCULATION OF L.E. :

Born Haber Cycle for NaCl (s)


1 Cl (g) Hf
Na (s) + 2 NaCl(s)
2
Hsub 1( H )
BE
2
Na(g) HL.E.
Cl(g)
HI.E. HEg
+ -
Na (g) + Cl (g)

Hf = Hsub + H.E. + HBE + Heg + HL.E.

SOLVATION OR HYDRATION :

Whenever any compound is dissolved in an polar solvent then, different ions of the compound will
get separated and will get surrounded by polar solvent molecules. This process is known as
solvation or hydration. Energy released in this process is known as solvation energy or hydration
energy.
The ionic compound will be soluble only if solvation energy (H.E.) is more than the lattice energy.

FACTORS AFFECTING SOLVTION ENERGY OR HYDRATION ENERGY.


 1 1 
S.E.  Z+ Z–    
r  r 

CatalyseR Eduventures (India) Pvt. Ltd.


CHEMICAL BONDING 45

PROPERTIES OF IONIC COMPOUNDS


 Ionic bonds are Non directional in nature


 High Melting points / Boiling points.
 In solid state they are bad conductor (due to absence of charge carrier) while in aqueous &
molten state they are good conductor of electricity.
 Soluble in Polar solvents
 Show isomorphism.

CAVALENT BOND

(i) A covalent bond is formed by mutual sharing of one or more electron pairs between two
atoms to aquire nearest noble gas configuration.

or Cl – Cl
8e– 8e–

Sharing of one e–  Single bond


Sharing of two e –
 Double bond
Sharing of three e –
 Triple bond

Variable covalency: Shown by elements having vacant ‘d’ orbitals (caused due to excitation of the
electron.)

8e
Cl
Cl C Cl
– –
H O H 8e Cl 8e

– 8e–
2e 8e– 2e– Each of the four Cl atoms
H atom attain a duplet of along with the C atom attains
electrons and O , the octet octet of electrons

O C O or O C O

8e– 8e– 8e–


Double bonds in CO2 molecule

PROPERTIES:

 Covalent bonds are directional in nature


 Low melting point & boiling point. (except Diamond / Graphite, due to their peculiear structure)
 Electrical conductivity due to auto-protolysis or self ionisation.
Show isomerism.
Soluble in nonpolar solvents

Nothing is impossible, the word itself says ‘I’m possible’!


46 CHEMICAL BONDING

Co-ordinate bond or dative bond is special type of covalent bond is which shared bond pair of
electron is donated by one atom, called donor and other atom is acceptor.
+  or [ A  B]

 H F 
H  
F
 | | 
| |
H– N: + B–F  H – N  B – F 
| |
H  
F
 | | 
 H F 
 
NH 3 + H + = NH 3  H + or NH 4 +

CHARACTERISTICS OF COORDINATE COMPOUNDS

a) Their melting and boiling points are higher than purely covalent compounds and
lower than purely ionic compounds
b) Soluble in non-polar solvents
c) They are as stable as the covalent compounds.
d) They are bad conductors of electricity.
e) The bond is rigid and directional & show isomerism.

LEWIS STRUCTURES
Step I. Calculated the total number of valence electrons.

Step II. For anion, add number of electrons equal to the units of ve charge and for cation,
subtract number of electrons equal to the units of +ve charge.

Step III. Select the central atom (least electronegative atom) and draw the skeletal structure. H
and F usually occupy terminal position.

Step IV. Put one shared pair of electrons between every two atoms. Use the remaining pairs of
electrons either for multiple bonding or to show them as lone pairs, keeping in mind that
octet of each atom is completed.
Formal Charge
The formal charge of an atom in a polyatomic molecule or ion may be defined as the difference
between the number of valence electrons of that atom in an isolated or free state and the number
of electrons assigned to that atom in the Lewis structure. It is expressed as :

Formal charges help in the selection of the lowest energy structure from a number of
possible Lewis structures for a given species.

CatalyseR Eduventures (India) Pvt. Ltd.


CHEMICAL BONDING 47

MODERN CONCEPT OF COVALENT BOND (VBT) :

A covalent bond is formed by the partial overlap of two half filled atomic orbitals containing
electrons with opposite spin. The extent of overlap decides the strength of a covalent bond.

< < [Principal Quantum no. same, n = 2]


  
 s orbital are spherical in nature so they are least diffused hence it will provide less area for
overlapping.

TYPES OF OVERLAPPING

+ + +  + +

s s s-s  bond

+ + + -  + + -

s px s-px  bond

- + + + -  - + + -
px px px–px  - bond
+ + + +
+ 
- - - -
p  - p  bond

Table
SIGMA    BOND PI    BOND
(i) Formed by overlap of orbitals along their (i) Formed sideway overlapping of orbitals
internuclear axis (end to end overlap). (lateral overlapping)
(ii) Formed by overlapping between s-s, s-p or p-p (ii) Formed by the overlap of p-p orbitals only.
orbitals.
(iii) Overlapping is quite large and hence sigma (iii) Overlapping is to a small extent. Hence,  
bond is a strong bond. bond is a weak bond.
(iv) Electron could is symmetrical about the line (iv) Electron cloud is unsymmetrical.
joining the two nuclei.
(v) Consists of only one electron cloud, (v) Consists of two electron clouds, one above
symmetrical about the internuclear axis. the plane of atomic nuclei and the other below
it.
(vi) Free rotation about a   bond is possible. (vi) Free rotation about a   bond is not possible
because on rotation, overlapping vanishes
and so the bond breaks.

Nothing is impossible, the word itself says ‘I’m possible’!


48 CHEMICAL BONDING

BOND PARAMETERS

1. Bond Length
The average distance between the centres of the nuclei of the two bonded atoms is called its
bond length.
Factors Affecting Bond Length :
(i) Size of the atoms varies directly. (ii) Multiplicity of bonds varies inversly.
(iii) Type of hybridisation - Greater the s-character, shorter is the hybrid orbital and hence
shorter is the bond length.

2. Bond Enthalpy (Bond Energy)


The amount of energy required to break one mole of bonds of a particular type so as to separate
them into gaseous atoms is called bond dissociation enthalpy or simply bond enthalpy.
Factors Affecting Bond Enthalpy :
(i) Size of the atoms varies inversly (ii) Multiplicity of bonds varies directly.
(iii) Number of lone pairs of electrons present C  C  N  N  O  O  F  F

3. Bond Angle
The angle between the lines representing the directions of the bonds, i.e., the orbitals containing
the bonding electrons is called the bond angle.

4. Bond Order
In the Lewis representation of a molecule or ion, the number of bonds present between two
atoms is called bond order.
Isoelectronic species have the same bond order, i.e.
(i) Greater the bond order, greater is the stability of the bond, i.e., greater is the bond
enthalpy.
(ii) Greater the bond order, shorter is the bond length.

DEVIATION FROM IDEAL BEHAVIOR

COVALENT CHARACTER IN IONIC COMPOUNDS (FAJAN’S RULE) :

 There is no compound which is 100% ionic.

 Covalent character in ionic compound can be explained with the help of Fajan’s rule.
According to Fajan’s rules, covalent character will be more if
Cation Anion
(i) Small size (i) Large size
(ii) More charge (ii) More charge
(iii) Pseudo inert gas configuration of cation

More distortion of anion, more will be polarisation then covalent character increases.

CatalyseR Eduventures (India) Pvt. Ltd.


CHEMICAL BONDING 49

PARTIAL IONIC CHARACTER OF COVALENT BOND


(DIPOLE MOMENT)
Dipole moment is a vector quantity    q  d. Units = coulomb m (S.I.) or esu cm (CGS) or
Debye (common unit) 1D  10 18 esu cm  3.33  10 30 coulomb m

DIPOLE MOMENT

 
A B
X
Let B is more –ve than A.
Resultant DM

P2  Q2  2PQ cos  

observed D.M.
% ionic character = × 100 %
calculated D.M. (for 100 % ionic)

Dipole moment depends on

 Electronegativity difference between bonded atoms


 Direction of bond dipole moment
 Angle between various bonds
 Influence of unshared e– pairs
 Magnitude of polarity of the molecule
 Symmetrical / Unsymmetrical shape.

VSEPR Theory (Valence Shell Electron Pair Repulsion Theory)


The electron pairs surrounding the central atom repel one another and more so far apart
from one another that there are no further repulsions between them. As a result, the molecule has
minimum energy and maximum stability.
The order of repulsion between electron pairs is as follows:
Lone pair – Lone pair > Lone pair – Bond pair > Bond pair – Bond pair

Hybridisation
The orientations of hybrid orbitals generated will be dependent on type of atomic orbitals and on
number of atomic orbitals taking part in hybridisation
s+p  2 new sp hybridised orbitals (sp hybridisation)
s +2p  3 new sp2 hybridised (sp2 hybridisation)

Nothing is impossible, the word itself says ‘I’m possible’!


50 CHEMICAL BONDING

s + 3p  4 new sp3 hybridised


s + 3p + d  sp3d
s + 3p + 2d  sp3d2
s + 3p + 3d  sp3 d3
d + s + 2p  dsp2

Types of No. of No. of Shape of


Molecule / bond lone Hybrid Orbitals Molecule/ Examples
Ion pairs pairs Ion

2 0 sp – linear Linear BeCl2, HgCl2

2 1 sp2 –Trigonal planar bent (V-shape) SnCl2, PbCl2


AX2
2 2 sp3 – Tetrahedral bent (V-shape) H2O, OF2

2 3 sp3d – Trigonal bipyramidal Linear I3–, XeF2

3 0 sp3 – Trigonal planar Trigonal planar BF3, BCl3

AX3 3 1 sp3 – Tetrahedral Pyramidal NH3, H3O+

3 2 sp3d – Trigonal bipyramidal T-shape ClF3, BrF3

4 0 sp3 – Tetrahedral Tetrahedral CH4, NH4+


Distorted
AX4 4 1 sp3d – Trigonal pyramidal SF4
tetrahedral
4 2 sp3d2 – Octahedral Square planar XeF4
Trigonal
5 0 sp3d – Trigonal bipyramidal PCl5
AX5 bipyramidal
5 1 sp3d2 – Octahedral Square pyramidal IF5

6 0 sp3d2 – octahedral Octahedral SF6


AX6 sp3d3 – pentagonal Distorted
6 1 XeF6
bipyramidal Octahedral
3 3
sp d – pentagonal Pentagonal
AX7 7 0 IF7
bipyramidal bipyramidal

CatalyseR Eduventures (India) Pvt. Ltd.


CHEMICAL BONDING 51

PREDICTING THE HYBRID STATE OF THE CENTRAL ATOM IN COVALENT MOLECULES OF


POLYATOMIC ATOM/IONS

1
Hybrid State 
2
 V  M  c  a
V  Valence e  in central atom M  Monovalent atom attached to central atom
c  Magnitude of positive charge on cation a  Magnitude of Negative charge on anion

RESONANCE
Resonance may be defined as the phenomenon in which two or more structures involving in
identical position of atom, can be written for a particular compound
– –
O O O
C C C
O O O O O O
Resonance Hybrid:- It is the actual structure of all different possible structures that can be
written for the molecule without violating the rules of covalence maxima for the atoms.

Rules for Resonance


(A) All the canonical forms must have proper lewis structure.
(B) The positions of the nuclei of the atoms must remain the same in all of the structures.
(C) All atoms taking part in the resonance must lie in a plane.
(D) The energy of actual molecule is lower than of any form.
(E) The more stable a structure is the greater contributor to its resonance hybrid.

MOLECULAR ORBITAL THEORY :

The molecular orbital theory was developed by F. Hund and R.S. Mulliken in 1932. The salient
features are:
(i) Molecular orbitals are formed by the combination of atomic orbitals of comparable energies and
proper symmetry.
(ii) A molecular orbital is influenced by two or more nuclie depending upon the number of the atoms
in the molecule.
(iii) Two atomic orbitals combine, two molecular orbitals called bonding molecular orbital and anti-
bonding molecular orbital are formed.
ABMO

Energy
Atomic
Atomic orbital
orbital
BMO

(iv) The bonding molecular orbital has lower energy and hence greater stability than the
corresponding antibonding molecular orbital.
(v) The molecular orbitals like the atomic orbitals are filled in accordance with the Aufbau principle
obeying the Pauli Exclusion principle and the Hund’s Rule of Maximum Multiplicity.

Nothing is impossible, the word itself says ‘I’m possible’!


52 CHEMICAL BONDING

ENERGY LEVEL DIAGRAM FOR MOLECULAR ORBITALS :

The increasing order of energies of various molecular orbitals for O2 and F2 is given below
 1s < *1s < 2s < *2s < 2Pz < (2px =2py) < *2px = *2pz < *2pz .

For molecules such as B2, C2, N2 etc. the increasing order of energies of various molecular
orbitals is
1s < * 1s < 2s < *2s < (2Px = 2py) < 2pz < (*2Px = *2py) < *2pz

BOND ORDER
Bond order (B.O.) is defined as one half the difference between the number of electrons
present in the bonding and the antibonding orbitals
i.e., Bond Order (B.O.) = ½ (Nb – Na)
Nb  number of electrons in bonding molecular orbitals.
Na  number of electrons in non-bonding molecular orbitals.

FORCES OF ATTRACTION (WEAKER BONDS)

 Hydrogen bonding: When a hydrogen atom is bonded to a highly electronegative atom (like F,
O or N) comes under the influence of another strongly electronegative atom, then a weak
electrostatic force of attraction is developed between them, which is called as hydrogen bond.

Conditions for Hydrogen Bonding


1. Hydrogen should be linked to a highly electronegative element.
2. The size of the electronegative element must be small.
These two criterias are fulfilled by F, O, and N in the periodic table.

TYPES OF HYDROGEN BONDING


Intermolecular hydrogen bonding: This type of bonding takes place between two
molecules of the same or different types. For example,
H H H

O—H----- O — H ------ O — H ------

Intramolecular hydrogen bonding: This type of bonding occurs between atoms of the
same molecule present on different sites.
H H
O O O
O
N C
O H

Salicaldehyde
o-nitrophenol

CatalyseR Eduventures (India) Pvt. Ltd.


CHEMICAL BONDING 53

APPLICATIONS IN:
(A) Abnormal behaviour of water.
(B) Association of molecules eg. association of CH3COOH, HCOOH
(C) Dissociation of a polar species.
(D) Abnormal melting point & boiling point. (E) Enhanced solubility in water.

INTERMOLECULAR FORCES (VAN DER WAAL’S FORCES) :

Intermolecular attractions hold two or more molecules together. These are weakest chemical
forces and can be of following types.

(A) Ion-dipole attraction :


Exists between an ion and a polar molecule. Its strength depends on (i) size of ion (ii) charge on
the ion (iii) dipole moment of the polar molecule.

H
 This force is responsible for hydration.
Na  O


H

(B) Dipole-dipole attraction :


Electrostatic attractions between the oppositively charged ends of permanent dipoles. Exists
between polar molecules and due to this force gas can be liquified.
   
H  Cl H  Cl
(C) Ion-induced dipole attraction :
Exists between ion and non-polar molecules
 
Na Cl  Cl
(D) Dipole-induced dipole attraction :
Exists between polar and non-polar molecules.
   
H  Cl Cl  Cl
(E) Instantaneous dipole- Instantaneous induced dipole attraction :
Exists among the non-polar molecules like H2, O2, Cl2 etc. in solid or liquid states

Metallic bonds: Electron gas model or sea model, with metal atom existing as kernels along with
less firmly held valence e  s & bonds between various kernels ( at the lattice site) & valence e  s
is known as metallic bonds.

Nothing is impossible, the word itself says ‘I’m possible’!


54 CHEMICAL BONDING

SOME TYPICAL BOND

1. Back Bonding
If among the bonded atoms, one atom has a vacant orbital & another has excess of e  s
then a sort of  bonding takes place between the two. If this is between ‘P’ orbitals of the two,
this is known as p  p back bonding.
Most efficient when the atoms are very small & the orbitals involved of the two are of
same energy level.

2. Banana Bond
This type of bonding is present in B2H6 .
This structure shows that there are two types of hydrogen atom-Terminals and bridging.
H H H

122 B 97 B

H H
H
1.77 A




CatalyseR Eduventures (India) Pvt. Ltd.


CHEMICAL BONDING 55

SOLVED EXAMPLES

1. The bond angle of H2O is 104° while that that of F2O is 102°. Explain
Solution: Both H2O and F2O have a lone pair of
electrons. But fluorine being highly
electronegative, the bond pair electrons
O O
repulsion more
are drawn more towards F in F2O,
whereas in H2O it is drawn towards O. So H H F F
in F2O the bond pairs being displaced Repulsion less
away from the central atom, has very little tendency to open up the angle. But in H2O
this opening up is more as the bond pair electrons are closer to each othler. So bond
 of F2O is less than H2O.

2. Na2CO3 does not decompose on heating whereas CaCO3 decomposes, why?


Solution: The solution lies in the thermal stability of Na2CO3 compared to CaCO3. The
decomposition of a carbonate means formation of an oxide along with CO2
CaCO3  CaO + CO2 (g)
Now more the carbonate ion is polarised by the cation more is the chance of formation of
CO2 and therefore higher is the probability of decomposition. Now let’s see the case of
CaCO3. The polarising power of Ca2+ is more than Na+ due to higher cationic charge of
calcium ion. Therefore it polarises the carbonate ion more compared to sodium whose 
is insufficient to polarise CO23  and so CaCO3 decomposes but not Na2CO3.

O
++ C
Ca

O O   CaO  CO2

O 
  No polarization
+
Na C

O O

3. Dipole moment of CCl4 is zero while that of CHCl3 is non zero.


Solution: Both CCl4 & CHCl3 have tetrahedral structure but CCl4 is symmetrical while CHCl3 is non-
symmetrical.
Cl H

C C

Cl Cl Cl Cl
Cl Cl
Symmetrical Non-Symmetrical

Due to the symmetrical structure of CCl4 the resultant of bond dipoles comes out to be
zero. But in case of CHCl3 it is not possible as the presence of hydrogen introduces some
dissymmetry.

Nothing is impossible, the word itself says ‘I’m possible’!


56 CHEMICAL BONDING

4. Both CO2 & N2O are linear, but dipole moment of CO2 is zero but N2O is non-zero.
Why?
Solution: The answer lies in the structure of these molecules. While CO2 is symmetrical, N2O is
not, for which the bond dipoles do not cancel each other leaving the molecule with a
resultant dipole.

NNO

–29
5. Dipole moment of KCl is 3.336  10 coulomb metre which indicates that it is
highly polar molecule. The interatomic distance between K+ and Cl– is 2.6  10–10m.
Calculate the dipole moment of KCl molecule if there were opposite charges of one
fundamental unit located at each nucleus. Calculate the percentage ionic character
of KCl.
Solution: Dipole moment  = e  d coulomb metre For KCl d = 2.6  10–10 m
For complete separation of unit charge e = 1.602  10–19 C
Hence  = 1.602  10–19  2.6  10–10 = 4.1652  10–29 Cm
KCl = 3.336  10–29 Cm
3.336  10 29
 % ionic character of KCl =  100 = 80.09%
4.165  10 29

6. o-hydroxy benzaldehyde is less soluble in water than p-hydroxy benzaldehyde.


Explain.
Solution: A substance is said to be soluble in water if it is capable of forming H-bonding with water
molecule. In o-hydroxy benzaldehyde due to intra-molecular chelation the –OH group is
not available to form hydrogen bond with water hence it is sparingly soluble in water. On
the other hand the –OH group is available in p-hydroxy benzaldehyde to form H-bond
with water and hence it is more soluble compared to the o-isomer.
O H O H O H
H

O
C

H
o-Isomer H C O p-isomer

7. HF forms stronger H-bonds than H2O, still  Hvap of HF is lower than that of pure
water. Explain.
Solution: The number of H–bonds broken per mole of HF on vapourisation is much less than in
the case of H2O. Each HF forms two bonds, while each H2O molecule forms 4 bonds. In
case of water vapours at boiling pt. contains essentially monomers while HF contains
polymers i.e., all H–bonds are not broken on vapourisation whereas all 4-H bonds are
broken per molecule of water.

CatalyseR Eduventures (India) Pvt. Ltd.


CHEMICAL BONDING 57

8. Compare the stabilities of H2, H2+ and H2–.


1
Solution: Configuration of H2 will be 1s2, 1s0 B.O. = (2 – 0) = 1
2
1 1
H2+ means removal of an electron from the bonding M.O. B.O. = (1 – 0) =
2 2
1 1
H2– means addition of an electron in the anti-bonding M.O. (1s).  B.O. = (2 – 1) =
2 2

So from the bond order it may be seen that both H2 & H2 may have the same bond
energy. But removal of an electron from a diatomic species tend to decrease the inter
electronic repulsion and thereby shorters the bond length. So the bond energy
becomes more than that compared to H2 .
Higher the bond order the greater will be the bond energies and hence greater the
stability.  H2 > H2 > H2

9. Why is that Lithium salts have a greater degree of covalent character than other
halides of the group?
Solution: The small size of Li+ gives it a large polarising power, hence covalent nature of its
compound increases. It can be viewed in terms of Fajan's rule
 Smaller the cation
 Larger the anion
 Greater the charge density as the surface causes greater polarising power of Li+
Hence, greater the covalent nature.

10. The solubility of the hydroxides of the alkaline earth metals increases i.e. Ba(OH) 2
has got a higher solubility in water compared to Mg(OH)2-. Why?
Solution: Here both the cations Ba2+ and Mg2+ have the same charge, but as the radius of Ba2+ is
more therefore  of Ba2+ is less which implies that Ba(OH)2 having higher degree of ionic
character is more soluble in polar solvents like water.
But now if I ask to predict the solubility of MgSO4 & BaSO4. The answer seems
to be quite similar to the earlier one and BaSO4 turns out to be the one having higher
solubility. But contrary to our expectation the trend is reversed here. BaSO4 is sparingly
soluble in water. The question comes why? In case of hydroxide it is something. In case
of sulfate it’s the other way around. Is there any way by which we can a predict the
solubility trend? The answer is yes.
When a lattice is dissolved in water, the ions became solvated and the solvated
ions are more stable than a free ion and due to this stability energy is released. This
energy released is called solvation energy and if this overcomes the lattice energy then it
–1 +
is soluble. The lattice energy of NaCl is 778 kJ mol and the heats of hydration of Na
and Cl– is –787 kJ mol–1. As it is more than the lattice energy of NaCl therefore it is
soluble.

Nothing is impossible, the word itself says ‘I’m possible’!


58 CHEMICAL BONDING

Now we should focus our attention to the solubility trend in a given series. For a
comparison of the solubility both the lattice energy and hydration energy factors have to
be taken into account. If in a series the decrease of lattice energy is more compared to
the decrease in hydration energy then the substance becomes soluble.
k1 k 2
Now the hydration enthalpy of a salt is given by Hhydration   where k1 and
r r
k3
k2 are constants & lattice energy U  k3 = constant
r  r

Case (i): When r+ << r– the contribution of the anion to the hydration enthalpy is
small so the total Hhydration would be dominated by the cation alone. In a
series of salts of a large anion, the hydrational enthalpy will decrease in
magnitude with increasing cation size. Now how does the lattice energy
respond to this changing cation radius? The lattice energy is inversely
proportional to (r+ + r–). Since r– >> r+, the sum will not change
significantly as r+ increases. Consequently the lattice energy will not
decrease as fast as the hydration energy with increasing cationic size.
The more quickly diminishing hydration energy results in a decrease in
solubility.
E.g. Solubility of LiI > NaI > KI…
MgSO4 > CaSO4 > SrSo4 > BaSO4
Case (ii): r+  r–
Here the lattice energy decreases with increasing cationic size more
rapidly than the hydration energy which therefore results in an enhanced
solubility in a series.
E.g. Solubility of LiF < NaF < KF
Mg(OH)2 < Ca(OH)2 < Ba(OH)2

11. The melting point of NaCl is higher than that of AgCl. Explain
Solution: Now whenever any comparison is asked about the melting point of the compounds which
are fully ionic from the electron transfer concept it means that the compound having lower
melting point has got lesser amount of ionic character than the other one. To analyse
such a question first find out the difference between the 2 given compounds. Here in both
the compounds the anion is the same. So the deciding factor would be the cation. Now if
the anion is different, then the answer should be from the variation of the anion. Now in
the above example, the difference of the cation is their electronic configuration. Na+ =
[Ne]; Ag+ = [Kr] 4d10. This is now a comparison between a noble gas core and pseudo
noble gas core, the analysis of which we have already done. So try to finish off this
answer.

CatalyseR Eduventures (India) Pvt. Ltd.


CHEMICAL BONDING 59

12. Which of the following molecules are polar: H2O, NH3,SO2,CHCl3,BF3?


Solution: Cl F

O C B
O N O F
S H H F
H H H O=C=O
H H H
Non-polar (Polar) Polar Non-polar
(Polar) (Polar)
 = 1.84D  = 1.86D =0  = 1.60D  = 1.86D =0

13. Compare the dipole moment of Cis 1,2 dichloroethylene and trans 1, 2
dichloroethylene.
Solution: Cl Cl Cl H

C C
C C
H Cl
H H
Trans
Cis

In the trans compound the C-Cl bond dipoles are equal and at the same time acting in
opposition cancel each other while Cis compound the dipoles do not cancel each other
resulting in a higher value.
Generally all trans compounds have a lower dipole moment compared to the
corresponding cis isomers, when both the substituents attached to carbon atom are
either electron releasing or electron withdrawing.

14. Explain why ethyl alcohol (C2H5OH) has got a higher boiling point than dimethyl
ether (CH3-O-CH3) although the molecular weight of both are same?
Solution: Though ethyl alcohol and dimethyl ether have the same molecular weight but in ethyl
alcohol the hydrogen of the O-H groups forms intermolecular hydrogen bonding with the
OH group in another molecule. But in case of ether the hydrogen is linked to C is not so
electronegative to encourage the hydrogen to from hydrogen bonding.
C2H5 C2H5
 
-----O  H ----------O  H ------
Due to intermolecular H-bonding, ethyl alcohol remains in the associated form and
therefore boils at a higher temperature compared to dimethyl ether.

Nothing is impossible, the word itself says ‘I’m possible’!


60 CHEMICAL BONDING

15. Explain why O-nitrophenol is more volatile than p-nitrophenol?


Solution: More volatility means it has got lower boiling point. Now p-nitrophenol remains associated
through intermolecular hydrogen bonding. But in o-nitrophenol only intramolecular H-
bonding formation takes places, as a result of which there is no association. So p-
nitrophenol which remains as an associated species has got higher boiling point and so
less volatile.
O H O H O H
H

O
N

o
O N O
o-Isomer
p-isomer

16. Draw the structure of the following


(i) SOF4 (ii) XeO3 (iii) POCl3
Solution: O ii) i ii) O
i)
F
F Xe
S
P
O O
O
Cl Cl
F F Cl

17. In o-dichlorobenzene the observed value of dipole moment is 6.00 D while it’s
theoretical value is 6.30 D why?
Solution: o-dichlorobenzene due to dipole-dipole repulsion bond Cl
angle increase. Therefore dipole moment decreases.
Cl

18. Dipole moment of o-chlorphenol is less than o-fluoro-phenol. Explain


Solution: Cl F H
OH O

no H-bonding

Due to H-bonding on o-fluorophenol bond angle decrease ( 60°) hence dipole moment
is increased.

19. BiCl5 does not exist. Explain


Solution: Due to inert pair effect, stability of Bi3+  Sb3+  As3+ and Bi5+  Sb5+  As5+ .
Hence Bi5+ salts are least stable; so BiCl5 does not exist.

CatalyseR Eduventures (India) Pvt. Ltd.


CHEMICAL BONDING 61

20. Dipole moment of BF3 is zero. Explain


Solution: BF3 is a planar molecule with the bond angles being 120°.
In each B-F bond the bond dipole is projected towards
Fluorine. Now the direction of the resultant bond dipoles of F
these two bonds is shown in the diagram as R. If R is equal
to the dipole of the 3rd B-F bond then R and B-F dipole will
R
neutralize each other as equal dipoles acting in opposite
B
direction cancel each other. Now let’s see the magnitude of
R. As dipole moment is a vector quantity, so the resultant of
F 120°
two dipoles can be obtained from the law of vector addition. F
Suppose the bond dipole is 1.

R = a2  b2  2abcos  a & b are the individual vectors

= 12  12  211 cos120  = angle between vectors

 1  1
= 212  212    cos120° =   
 2  2

= 212  12 = 1 = B–F

 R is equal in magnitude to B-F bond dipole. Similar is the case with the other two
resultants. So the net dipole of BF3 is zero.

21. Carbon has a co-ordination number of four while other elements of this group have
co-ordination number of six. Why?
Solution: The normal valency of the elements is four, but apart from carbon, the elements can
make more than four bonds. This because they make use of a set of d-orbitals in
bonding. The availability of d-orbitals empty but electrons of ns2 and np2 can be excited
into this is responsible for the ability of the elements, except carbon, to make complex
ions such as SiF62









Nothing is impossible, the word itself says ‘I’m possible’!


62 CHEMICAL BONDING



EXERCISE # 01 OBJECTIVE EXERCISE (LEVEL # 01)


BASED ON IONIC BOND

1. Chemical formula for calcium pyrophosphate is Ca2P2 O7 The formula for ferric pyrophosphate will
be
(A) Fe3 (P2O7 )3 (B) Fe 4P4 O14 (C) Fe4 (P2 O7 )3 (D) Fe3PO 4

2. Which of the following is least soluble


(A) BeF2 (B) SrF2 (C) CaF2 (D) MgF2

3. Which of the following halides has maximum melting point


(A) NaCl (B) NaBr (C) NaI (D) NaF

BASED ON COVALENT BOND

4. The most covalent halide is:


(A) AlF3 (B) AlCl3 (C) AlBr3 (D) All3

5. A bond with maximum covalent character between non-metallic elements is formed


(A) Between identical atoms
(B) Between chemically similar atoms
(C) Between atoms of widely different electronegativities
(D) Between atoms of the same size

6. The correct sequence of increasing covalent character is represented by


(A) LiCl  NaCl  BeCl2 (B) BeCl2  NaCl  LiCl
(C) NaCl  LiCl  BeCl2 (D) BeCl2  LiCl  NaCl
7. The covalency of nitrogen in HNO3 is
(A) 0 (B) 3 (C) 4 (D) 5

8. As compared to covalent compounds, electrovalent compounds generally have


(A) Low melting points and low boiling points
(B) Low melting points and high boiling points
(C) High melting points and low boiling points
(D) High melting points and high boiling points

BASED ON OVERLAPING -  AND - BONDS

9. The number of sigma and pi bonds in 1-butene-3-yne are


(A) 5 sigma and 5 pi (B) 7 sigma and 3 pi
(C) 8 sigma and 2 pi (D) 6 sigma and 4 pi

10. In which of following there exists a p  d bonding


(A) Diamond (B) Graphite (C) Dimethyl amine (D) Trisilylamine

CatalyseR Eduventures (India) Pvt. Ltd.


CHEMICAL BONDING 63

11. Number of bonds in SO2


(A) Two  and two  (B) Two  and one 
(C) Two , two  and one lone pair (D) None of these

12. Number of sigma bonds in P4 O10 is


(A) 6 (B) 7 (C) 17 (D) 16

BASED ON CO-ORDINATE OR DATIVE BONDING

13. The compound containing co-ordinate bond is


(A) O3 (B) SO3 (C) H2SO4 (D) All of these

14. The number of dative bonds in sulphuric acid molecules is


(A) 0 (B) 1 (C) 2 (D) 4

15. The structure of orthophosphoric acid is


O
H
 |
(A) HO  P O H (B) O  P OH
| |
O O
| |
H H
O
H
| 
(C) O  P O H (D) HO P  O
|
H

16. What is the nature of the bond between B and O in (C2H5 )2 OBH3
(A) Covalent (B) Co-ordinate covalent
(C) Ionic bond (D) Banana shaped bond

BASED ON HYBRIDIZATION

17. A square planar complex is formed by hybridisation of which atomic orbitals


(A) s, px , py , dyz (B) s, p x , p y , dx2  y2 (C) s, p x , p y, dz2 (D) s, py , pz ,dxy

18. The percentage s-character of the hybrid orbitals in methane, ethene and ethyne are respectively
(A) 25, 33, 50 (B) 25, 50, 75 (C) 50, 75, 100 (D) 10, 20, 40

19. In a regular octahedral molecule, MX6 , the number X  M  X bonds at 180° is


(A) Six (B) Four (C) Three (D) Two

20. The correct order of bond angles (smallest first) in H2 S,NH3 ,BF3 and SiH4 is
(A) H2 S  NH3  SiH4  BF3 (B) NH3  H2 S  SiH4  BF3
(C) H2 S  SiH4  NH3  BF3 (D) H2 S  NH3  BF3  SiH4

Nothing is impossible, the word itself says ‘I’m possible’!


64 CHEMICAL BONDING


21. The geometrical arrangement and shape of I3 are respectively
(A) Trigonal bipyramidal, T-shape (B) Octahedral, linear
(C) Trigonal planar, Angular (D) Trigonal bipyramidal, linear

22. The molecule which possesses both sp3 and sp3d2 hybridisation is
(A) solid PCl5 (B) gaseous PCl5
(C) PCl4 (D) PCl6

BASED ON POLARISATION AND FAJAN’S RULE

23. Two electrons of one atom A and two electrons of another atom B are utilized to form a
compound AB. This is an example of
(A) Polar covalent bond (B) Non-polar covalent bond
(C) Polar bond (D) Dative bond

24. In which of the following molecule is the covalent bond most polar
(A) HI (B) HBr (C) HCl (D) H2
25. According to Fajan’s rule, covalent bond is favoured by
(A) Large cation and small anion (B) Large cation and large anion
(C) Small cation and large anion (D) Small cation and small anion
26. The correct order of decreasing polarisability of ion is:
(A) Cl ,Br  ,I ,F  (B) F , I , Br  , Cl (C) I , Br  , Cl , F (D) F , Cl ,Br  , I

27. Amongst LiCl, RbCl, BeCl2 and MgCl2 the compounds with the greatest and the least ionic
character, respectively, are
(A) LiCl and RbCl (B) RbCl and BeCl2 (C) RbCl and MgCl2 (D) MgCl2 and BeCl2

BASED ON DIPOLE MOMENT

28. The molecule which has the largest dipole moment amongst the following
(A) CH4 (B) CHCl3 (C) CCl4 (D) CHI3

29. Which of the following is the most polar


(A) CCl4 (B) CHCl3 (C) CH3 OH (D) CH3 Cl
30. Fluorine is more electronegative than either boron or phosphorus. What conclusion can be drawn
from the fact that BF3 has no dipole moment but PF3 does
(A) BF3 is not spherically symmetrical but PF3 is
(B) BF3 molecule must be linear
(C) The atomic radius of P is larger than the atomic radius of B
(D) The BF3 molecule must be planar triangular

31. Which molecule does not show zero dipole moment


(A) BF3 (B) NH3 (C) CCl4 (D) CH4

CatalyseR Eduventures (India) Pvt. Ltd.


CHEMICAL BONDING 65

32. N2 is less reactive than CN due to


(A) Presence of more electrons in orbitals (B) Absence of dipole moment
(C) Difference in spin quantum no (D) None of these

33. Which bond angle () would result in the maximum dipole moment for the triatomic molecule
XY2?
(A)  = 90° (B)  = 120° (C)  = 150° (D)  = 180°

34. The electronegativity difference between two atoms A and B is 2 then percentage of covalent
character in the molecule is
(A) 54% (B) 46% (C) 23% (D) 72%

BASED ON HYDROGEN BOND

35. Ethanol and dimethyl ether form a pair of functional isomers. The boiling point of ethanol is higher
than that of dimethyl ether due to the presence of
(A) Hydrogen bonding in ethanol (B) Hydrogen bonding in dimethyl ether
(C) CH3 group in ethanol (D) CH3 group in dimethyl ether

36. H2 O is a liquid while H2S is gas due to


(A) Covalent bonding (B) Molecular attraction
(C) H – bonding (D) H – bonding and molecular attraction

37. Select the compound from the following which dissolves in water
(A) CCl4 (B) CS2 (C) CHCl3 (D) C2H5 OH

BASED ON TYPES OF BONDING AND FORCES IN SOLID

38. The enhanced force of cohesion in metals is due to


(A) The covalent linkages between atoms
(B) The electrovalent linkages between atoms
(C) The lack of exchange of valency electrons
(D) The exchange energy of mobile electrons

39. Which shows the highest lattice energy?


(A) RbF (B) CsF (C) NaF (D) KF

40. In the formation of a molecule by an atom


(A) Attractive forces operate (B) Repulsive forces operate
(C) Both attractive and repulsive forces operate
(D) None of these

41. Which of the following statements is true about  Cu  NH3  4  SO 4


(A) It has coordinate and covalent bonds (B) It has only coordinate bonds
(C) It has only electrovalent bonds
(D) It has electrovalent, covalent and coordinate bonds

Nothing is impossible, the word itself says ‘I’m possible’!


66 CHEMICAL BONDING

BASED ON MOLECULAR ORBITAL THEORY

42. In PO34 ion, the formal charge on each oxygen atom and P  O bond order respectively are
(A) 0.75, 1.25 (B) 0.75, 1.0 (C) 0.75, 0.6 (D) 3, 1.25

43. According to molecular orbital theory which of the following statement about the magnetic
character and bond order is correct regarding O2
(A) Paramagnetic and bond order< O 2 (B) Paramagnetic and bond order> O 2
(C) Dimagnetic and bond order< O 2 (D) Dimagnetic and bond order> O 2

44. The bond order in NO is 2.5 while that in NO  is 3. Which of the following statements is true for
these two species
(A) Bond length in NO  is equal to that in NO
(B) Bond length in NO is greater than in NO 
(C) Bond length in NO  is greater than in NO
(D) Bond length is unpredictable

45. Bond energies in NO,NO and NO  are such as


(A) NO   NO  NO  (B) NO  NO   NO  (C) NO   NO  NO  (D) NO   NO   NO

46. In O2 , O 2 and O2


2 molecular species, the total number of antibonding electrons respectively are

(A) 7, 6, 8 (B) 1, 0, 2 (C) 6, 6, 6 (D) 8, 6, 8

47. Among KO2, AlO2–, BaO2 and NO2+ unpaired electron is present in
(A) NO2+ and BaO2 (B) KO2 and AlO2–
(C) KO2 only (D) BaO2 only

BASED ON RESONANCE

48. In the cyanide ion, the formal negative charge is on


(A) C (B) N
(C) Both C and N (D) Resonate between C and N

49. The number of possible resonance structures for CO23 is


(A) 2 (B) 3 (C) 6 (D) 9
50. Resonance hybrid of nitrate ion is

 2/3
(A) 1/ 2
O ........ N ........ O 1/ 2 (B) O........N........O 2/3 (C)  1/3
O........N........O 1/3 (D) 2/ 3
O...... N ...... O2/ 3
  

   
O 1/ 2 O 2/3 O 1/3 O 2/ 3

51. Among the following which has resonating structure :


(A) BF3 (B) PCl5 (C) SF6 (D) IF7


CatalyseR Eduventures (India) Pvt. Ltd.


CHEMICAL BONDING 67

EXERCISE # 02 OBJECTIVE EXERCISE (LEVEL # 02)


BASED ON IONIC BOND

1. An ionic bond A B is most likely to be formed when :


(A) the ionization energy of A is high and the electron gain enthalpy of B is low
(B) the ionization energy of A is low and the electron gain enthalpy of B is high
(C) the ionization energy of A and the electron gain enthalpy of B both are high
(D) the ionization energy of A and the electron gain enthalpy of B both are low

2. The compound which contains ionic as well as covalent bonds is


(A) C2H4 Cl2 (B) CH3 l (C) KCN (D) H2 O2

3. The hydration of ionic compounds involves :


(A) Evolution of heat (B) Weakening of attractive forces
(C) Dissociation into ions (D) All of these

4. In which of the following species the bonds are Non-directional?


(A) NCl3 (B) RbCl (C) BeCl2 (D) BCl3

5. Which has the lowest anion to cation size ratio :


(A) LiF (B) NaF (C) CsI (D) CsF

6. The compound which has the highest Lattice energy is


(A) LiF (B) LiCl (C) NaCl (D) MgO

7. A bond formed between two like atoms cannot be


(A) ionic (B) covalent (C) coordinate (D) metallic

8. Which of the following, when dissolved in water is non -conductor.


(A) Green Vitriol (B) Indian salt Petre
(C) Alcohol (D) Potash alum

9. An electrovalent compound does not exhibit space isomerism because of


(A) Presence of oppositively charged ions (B) High melting points
(C) Non-directional nature of the ionic bond (D) Crystalline nature

BASED ON COVALENT BOND, CO-ORDINATE BOND & LEWIS STRUCTURE

10. A sigma bond is formed by the overlap of atomic orbitals of atoms A and B. If the bond is formed
along the x-axis, which of the following overlaps is acceptable ?
(A) s orbital of A and pz orbital of B (B) p x orbital of A and p y orbital of B
(C) pz orbital of A and p x orbital of B (D) p x orbital of A and s orbital of B

11. PCl5 exists but NCl5 does not because :


(A) Nitrogen has no vacant 2d-orbitals (B) NCl5 is unstable
(C) Nitrogen atom is much smaller than P (D) Nitrogen is highly inert

Nothing is impossible, the word itself says ‘I’m possible’!


68 CHEMICAL BONDING

12. Which of the following has/have a strong covalent bond?


(A) Cl-F (B) F-F (C) C-Cl (D) C-F

13. Which of the following species are hypervalent?


1. PCl5 , 2. BF3 , 3. XeF2 , 4. CO23
(A) 1, 2, 3 (B) 1, 3 (C) 3, 4 (D) 1, 2

14. The types of bond present in N2 O5 are


(A) only covalent (B) only ionic
(C) ionic and covalent (D) covalent & coordinate

15. NH3 and BF3 combine readily because of the formation of :


(A) a covalent bond (B) a hydrogen bond (C) a coordinate bond (D) an ionic bond

16. Which of the following molecules does not have coordinate bonds?
(A) CH3  NC (B) CO (C) O3 (D) CO23

17. Which of the following Lewis dot diagrams is(are) incorrect ?

Cl
 
(A) Na  O  Cl  (B) Cl C Cl (C) (D)
 
Cl


 H  H H
 |  | |
(C) H  N  H [ S]2- (D) H  N N H
 |   

 H 
 2

18. The possible structure(s) of monothiocarbonate ion is :


2–
C C S S

S S C C
O O O O O O O O
(A) (B) (C) (D)

19. The total number of valence electrons in 4.2g of N3 ion are :
(A) 2.2 NA (B) 4.2 NA (C) 1.6 NA (D) 3.2 NA

BASED ON V. B. T. & HYBRIDISATION


1 2 3
20. In the following compound CH2  CH  CH2  C  CH , the C2  C3 bond is of the type :
(A) sp  sp2 (B) sp3  sp3 (C) sp  sp3 (D) sp2  sp3

21. Which of the following has a geometry different from the other three species (having the same
geometry)?
(A) BF4 (B) SO24 (C) XeF4 (D) PH4

CatalyseR Eduventures (India) Pvt. Ltd.


CHEMICAL BONDING 69

22. Maximum bond energy is in :


(A) F2 (B) N2 (C) O2 (D) equal

23. Number and type of bonds between two carbon atoms in CaC2 are :
(A) one sigma (  ) and one pi (  ) bond (B) one  and two  bonds
(C) one  and one and a half  bond (D) one  bond

24. In C  C bond in C2H6 undergoes heterolytic fission, the hybridisation of carbon in the resulting
two species is / are
(A) sp2 both (B) sp3 both (C) sp2 , sp3 (D) sp, sp2

25. The hybridisation and shape of BrF3 molecule are :

(A) sp3 d and bent T shape (B) sp2 d2 and tetragonal


(C) sp3 d and bent (D) none of these

26. The shape of methyl cation (CH3 ) is likely to be:


(A) linear (B) pyramidal (C) planar (D) spherical

27. The structure of XeF2 involves hybridization of the type :

(A) sp3 (B) dsp2 (C) sp3 d (D) sp3 d2

28. In the XeF4 molecule, the Xe atom is in the

(A) sp2-hybridized state (B) sp3-hybridised state


(C) sp3d3-hybridized state (D) sp3d2-hybridized state

29. Which of the following has been arranged in increasing order of size of the hybrid orbitals?
(A) sp  sp2  sp3 (B) sp3  sp2  sp (C) sp2  sp3  sp (D) sp2  sp  sp3

30. In the context of carbon, which of the following is arranged in the correct order of
electronegativity:
(A) sp  sp2  sp3 (B) sp3  sp2  sp
(C) sp2  sp  sp3 (D) sp3  sp  sp2

31. When 2s  2s, 2p  2p and 2p  2s orbitals overlap, the bond strength decreases in the order :
(A) pp  s s  p s (B) pp  ps  s s
(C) s s  pp  p s (D) s s  ps  pp

32. The shapes of IF5 and IF7 are respectively :


(A) distorted square pyramidal and pentagonal bipyramidal
(B) octahedral and pyramidal
(C) trigonal bipyramidal and pentagonal bipyramidal
(D) distorted square planar and distorted octahedral

Nothing is impossible, the word itself says ‘I’m possible’!


70 CHEMICAL BONDING

33. Carbon atoms in C2 (CN)4 are :


(A) sp-hybridized (B) sp2 -hybridized
(C) sp- and sp2 hybridized (D) sp, sp2 and sp3 - hybridized

34. CO2 is isostructural with


(I) HgCl2 (II) NO2 (III) SnCl4 (IV) C2H2
(A) I and III (B) II and IV (C) I and IV (D) III and IV

35. The ratio of  and  bonds in benzene is :


(A) 2 (B) 6 (C) 4 (D) 8

36. The bond angle and hybridization in ether (CH3 OCH3 ) is :

(A) 106º51, sp3 (B) 104º31, sp3 (C) > 109° 28' sp3 (D) None of these

37. The shape of a molecule which has 3 bond pairs and one lone pair is :
(A) Octahedral (B) Pyramidal (C) Triangular planar (D) Tetrahedral

38. Which molecule is T shaped :


(A) BeF2 (B) BCl3 (C) NH3 (D) ClF3

39. According to hybridisation theory maximum s-character is found in bond formed by (*) atom.
* * * *
(A) CH4 (B) SF6 (C) XeO46  (D) SF4

40. A   bond is formed by two p x orbitals each containing one unpaired electron when they
approach each other along :
(A) x - axis (B) y - axis (C) z - axis (D) any direction

41. Which of the following pairs is (are) isostructural?


(A) SF4 and SiF4 (B) SF6 and SiF62 (C) SiF62 and SeF62 (D) XeO46  and TeF62

42. The structure of XeF6 in vapour phase is


(A) pentagonal bipyramidal (B) trigonal bipyramidal
(C) capped octahedron (D) square bipyramidal

BASED ON MISSCELLEANEOUS (INCLUDING BOND ANGLES & BOND LENGTH)

43. Cyanogen, (CN)2 , has a ____ shape/structure :


(A) Linear (B) Zig-zag (C) V-shape (D) Cyclic

44. In which of the following sovents, KI has highest solubility? The dielectric constant  of each
liquid is given in parentheses.
(A) C 6H6  0  (B)  CH3 2 CO  2 
(C) CH3 OH  32  (D) CCl4  0 

CatalyseR Eduventures (India) Pvt. Ltd.


CHEMICAL BONDING 71

45. The formal charges on the three O-atoms in O 3 molecule are


(A) 0, 0, 0 (B) 0, 0, –1
(C) 0, 0, +1 (D) 0, +1, –1

46. The types of bonds present in CuSO4 . 5H2 O are


(A) electrovalent and covalent
(B) electrovalent, coordinate covalent & H-bond
(C) covalent, coordinate covalent & H-bonds
(D) electrovalent, covalent, coordinate covalent & H-bond

47. Which of the following has been arranged in order of decreasing dipole moment ?
(A) CH3 Cl  CH3F  CH3Br  CH3I (B) CH3F  CH3 Cl  CH3Br  CH3I
(C) CH3 Cl  CH3Br  CH3I  CH3F (D) CH3F  CH3 Cl  CH3I  CH3Br

48. Which of the following has the least dipole moment


(A) NF3 (B) CO2
(C) SO2 (D) NH3

Cl
49. The dipole moment of is 1.5 D. The dipole moment of is :

(A) 0D (B) 1.5 D


(C) 2.86 D (D) 2.25 D

50. The correct order of decreasing X  O  X bond angle is (X  H, F or Cl) :


(A) H2 O  Cl2 O  F2 O (B) Cl2O  H2 O  F2 O
(C) F2 O  Cl2O  H2 O (D) F2 O  H2O  Cl2 O

51. Which has higher bond energy :


(A) F2 (B) Cl2
(C) Br2 (D) I2

52. The bond angle in PH3 is :


(A) Much lesser than NH3 (B) Equal to that in NH3
(C) Much greater than in NH3 (D) Slightly more than in NH3

53. In the series ethane, ethylene and acetylene, the C  H bond energy is maximum in
(A) The same in all the three compounds (B) Ethane
(C) Ethylene (D) Acetylene

54. If ethylene molecule lies in X-Y plane then nodal planes of the   bond will lie in
(A) XZ plane (B) YZ plane
(C) In a plane that bisects C–C axis (D) XY plane

Nothing is impossible, the word itself says ‘I’m possible’!


72 CHEMICAL BONDING

BASED ON OTHER FORCES

55. Ethanol has a higher boiling point than dimethyl ether though they have the same molecular
weight. This is due to :
(A) resonance (B) coordinate bonding
(C) hydrogen bonding (D) ionic bonding

56. Arrange the following in order of decreasing boiling point :


(I) n-Butane (II) n-Butanol (III) n-Butyl chloride (IV) Isobutane
(A) IV  III  II  I (B) IV  II  III  I (C) I  II  III  IV (D) II  III  I  IV

57. For H2 O2 , H2 S, H2O and HF , the correct order of decreasing strength of hydrogen bonding is:
(A) H2 O  HF  H2 O2  H2 S (B) HF > H2O2 > H2O > H2S
(C) HF  H2 O  H2 O 2  H2 S (D) H2 O 2  H2 O  HF  H2 S

58. The order of strength of hydrogen bonds is:


(A) ClH...Cl  NH...N  OH...O  FH...F (B) ClH...Cl  NH...N  OH...O  FH...F
(C) ClH...Cl  NH...N  OH...O  FH...F (D) ClH...Cl  NH...N  OH...O  FH...F

59. Among the following, van der Waals forces are maximum in
(A) HBr (B) LiBr (C) LiCl (D) AgBr

60. The volatility of HF is low because of :


(A) its low polarizability (B) Strong intramolecular H-bonding
(C) its small molecular mass (D) Strong intermolecular H-bonding

61. Two ice cubes are pressed over each other and unite to form one cube. Which force is
responsible for holding them together :
(A) van der Waal’s forces (B) Covalent attraction
(C) Hydrogen bond formation (D) Dipole-dipole attraction

62. The pairs of bases in DNA are held together by :


(A) Hydrogen bonds (B) Ionic bonds
(C) Phosphate groups (D) Deoxyribose groups

63. In which of the following compounds, breaking of covalent bond takes place?
(A) Boiling of H2 O (B) Melting of KCN
(C) Boiling of CF4 (D) Melting of SiO 2



CatalyseR Eduventures (India) Pvt. Ltd.


CHEMICAL BONDING 73

EXERCISE # 03 OBJECTIVE EXERCISE (LEVEL # 03)


COMPREHENSION TYPE

Comprehension # 01 (Q.1 & Q.7)

According to molecular orbital theory in a molecule electrons are added in molecular orbitals in
order of their increasing energy. The number of electrons in the molecular orbitals is equal to the
sum of all the electrons on the bonding atoms. Like an atomic orbital each molecular orbital can
accommodate upto two electrons with opposite spins (in accordance with the Pauli exclusion
principle). When electrons are added to molecular orbitals of the same energy, the most stable
arrangement is predicted by Hund’s Rule; i.e., electrons enter these molecular orbitals singly with
parallel spins.
Constructive and destructive interaction between the wave functions of two hydrogen 1s orbitals
lead to the formation of a bonding (s) and an antibonding molecular orbitals ( *s ). In the bonding
molecular orbital, there is concentration of electron cloud between the nuclei of atoms which acts
as a negatively charged glue to hold the positively charged nuclei together. In the antibonding
molecular orbital there is a node between the nuclei that signifies zero electron density. However,
the energies of molecular orbitals actually increase as follow.
*
1s  1s   2s   *2s  2py  2pz   2p x  2p
*
y
*
 2p z
*
  2p x

This order is valid upto z  + 7  2. The energy of 2p x is lower in energy than the 2p y and 2pz

orbitals for O2 and F2. To evaluate the stabilities of species we determine their bond order,
defined as
1
Bond order = (No. of electron in B.M.Os – No. of electrons in ABMOs)
2
A bond order zero means the bond has no stability and the molecule cannot exist. The bond
order indicates the strength of a bond. Presence of unpaired electron in molecular orbitals make
the species paramagnetic, and the removal of electron from boding M.O is more difficult than that
of anti bonding M.O.

1. Which is not true for MOT?


(A) Some atomic orbitals keep their existence in a molecule while some convert into MOs.
(B) Molecular orbitals also follow Pauling’s exclusion principle and Hund’s rule besides aufbau
principle
(C) In the ABMO, the nuclei are repelled by each others positive charges rather than held
together.
(D) In a stable molecule, the number of electrons in B.M.Os is always greater than that in
ABMOs.

2. Among the following which is not paramagnetic?


(A) O2 (B) O2 (C) O2 (D) O22

Nothing is impossible, the word itself says ‘I’m possible’!


74 CHEMICAL BONDING

3. Which is false according to MOT


(A) H2 and He2 have same stability
(B) H2 is stable than H2
(C) He2 may exist while He2 cannot
(D) Same bond order of two species do not mean they have same bond energy

4. According to MOT
(A) In C2 molecule, there are three covalent bonds between two C atoms in which one is  and
other two are 
(B) C2 molecule does not exist
(C) In C2 molecule, there are two covalent bonds between two C atoms in which of one is 
and the other 
(D) In C2 molecule theer are two covalent bonds between two C atoms in which both are 

5. According to MOT the process N2(g)  N2 (g) + e is more endothermic than the process O2(g)
 O2 (g)  e because
(A) Electron is removed from anti bonding molecular orbital in N2 where as electron is removed
from bonding molecular orbital in O2
(B) N has half filled electronic configuration which is more stable
(C) Electron is removed from bonding molecular orbital in N2 where electron is removed from
anti bonding molecular orbital in O2
(D) Nitrogen is less electronegative than oxygen.

6. According to MOT, which is not true?


(A) The bond length of N2 is more compared with that for N2
(B) The bond length of O2 is less compared with that for O2
(C) When one electron is added to NO molecule bond length shortens
(D) When are electron is removed from NO molecule bond length shortens

7. Which species can exist among the following?


(A) B2 (B) Be2 (C) Ne2 (D) He2

Comprehension # 02 (Q.8 & Q.11)


The reducing effect on the nuclear charge by the inner electrons for an outer electron is termed
shielding (or screening). As a result of shielding, the outer electrons in an atom always
experience less nuclear charge than the actual charge Z. The effective nuclear charge (Z) as
experienced by an electron is then obtained by subtracting the total shielding contributions from
all the other electrons (i.e. except the one under consideration) from the actual nuclear charge.
Z* = Z – 
Where  = sum of the shielding contributions. The rules for estimating contributions to S are as
follows (Slater’s Rule)

CatalyseR Eduventures (India) Pvt. Ltd.


CHEMICAL BONDING 75

Contribution to shielding by each electron in

Electron All higher


Same group Groups n – 1 Groups  n – 2
Group group
1s 0 0.30 - -
(ns, np) 0 0.35 .85 1.00
(nd) or (nf) 0 0.35 1.00 1.00
for all groups on left, including s or p electrons of the same n.
According to Slater’s treatment, the energy of an electron is given by the empirical equation
2
 Z* 
E  13.6   eV Z = effective nuclear charge
 n 
The ionization energy of an atom or ion will be equal to the difference of energy between the
energies before and after ionization, e.g. for element M, the first ionization energy
I = E   EM
M

The E terms represent the sum of energies of all the electrons in the species.

8. Among the following which electron of Fe atom experience minimum attraction from nucleus?
(A) 3d (B) 4s (C) 2s (D) 2p
9. Z for a 1s electron in Fe atom is
(A) 25.15 (B) 25.7 (C) 25.65 (D) 25
10. I.P for Na (according to formula given) is
(A) Energy of its 3s electron (B) Energy of its 1s electron
(C) Energy of its 2s electron (D) Energy of its 2p electron
11. What is the energy of 4s electron of Fe atom
2 2 2 2
 3.75   26   13   6.25 
(A) 13.6    eV (B) 13.6    eV (C) 13.6    eV (D) 13.6    eV
 4   4   4   4 

Comprehension # 03 (Q.12 & Q.16)

Bond length is the average distance between the nuclei of the two atoms held by a bond. This
represents the internuclear distance corresponding to minimum potential energy for the system.
Main factors which affect the bond length are given below:—
i) Mulliple bonds are shorter than corresponding single bonds
ii) Sometimes single bond distances are some what larger than double of their respective
covalent radii (e.g. F2). It is due to strong repulsive interaction between the lone-pair electrons
on adjacent atoms.
iii) Some times single bond distances are some what shorter than double of their respective
covalent radii because bonds acquire some partial double bond character. This normally
happens when one atom having vacant orbital and another atom containing lone pair. It is
also possible it become shorter due to high ionic character in the covalent bond.

Nothing is impossible, the word itself says ‘I’m possible’!


76 CHEMICAL BONDING

12. Which is not true about the N – N bond length among the following speies

I. H2N — NH2 II. N2 III. HN— NH2 IV. N2O
(A) N — N bond length is shortest in II
(B) N — N bond length in I is shorter than that of in III
(C) N — N bond length in III is shorter than that of in I
(D) N — N bond length IV is intermediate between I and III

13. In which of the following case central atom-F bond has partial double bond character.
(A) NF3 (B) CF4 (C) PF3 (D) OF2

14. Which is correct about the C—F bond is


(A) C—F bond is shorter than expected because of lone pair-lone pair repulsion
(B) C—F is longer than expected because of lp — lp repulsion
(C) C—F is shorter than expected because of high ionic character
(D) C—F is longer than expected because of high ionic character

15. The correct order about of C—O bond length is


I. CO, II. CO2 III. CO23
(A) II  I  III (B) III  II  I (C) I  II  III (D) I  III  II

16. The bond angle is Cl2O is more than that in F2O because of so many reasons like
(A) p – d overlap in Cl2O which is not possible in F2O
(B) There is no lp – lp repulsion in Cl2O which is present in F2O
(C) F is more electronegative than oxygen while Cl is less electronegative than oxygen
The correct reason is (are)
(A) both a and b (B) only a (C) only b (D) all the three

Comprehension # 04 (Q.17 & Q.19)

The degree of polarity of a covalent bond is measured by the dipole moment (bond) of the bond
defined as
bond = charge on one of the poles  bond length
bond is a vector quantity. The dipole moment of a molecule is the vector addition of all the bond
dipole moments present in it. For a triatomic molecule containing two bonds like H2O, molecule is
2 2 2
given by molecule  bond  bond  2bond . bond cos   = bond angle
molecule is an experimental property so is also the bond length bond length is measured by
microwave spectroscopy which is based on the principle that absorption of microwave radiation
by a molecule causes rotational transition of molecules. However, a molecule with permanent
dipole moment equal to zero is micro-wave.
bond is experimental only when the molecule is diatomic.
The % ionic character of a bond is calculated using the equations

% ionic character =  obs  100
ionic
ionic = dipole moment when the molecule is completely ionic = 4.8 D per every A of bond length
–30
D = unit of dipole moment = 3.33  10 c.m.

CatalyseR Eduventures (India) Pvt. Ltd.


CHEMICAL BONDING 77

17. The bond length of a adiatomic molecule is 1.2 Å. The observed dipolemoment of the molecule is
1.15 D. Hence ionic character of the bond is nearly
(A) 16% (B) 15% (C) 20% (D) 30%

18. While going from H2O to H2 Se via H2S, the dipole moment of the molecule decreases because
(A) ionic character fo the bond between central atom and hydrogen decreases
(B) Bond angle increases
(C) Bond angle decreases
(D) ionic character increases and bond angle decreases

19. The correct order of the OO bond length in O 2 , H2 O2 and O 3 is :


(A) O3  H2 O2  O 2 (B) O3  H2 O2  O3
(C) O 2  O3  H2O 2 (D) H2 O2  O3  O2

Comprehension # 05 (Q.20 & Q.22)

3
As one moves from sp hybridisation to sp . % of s-character in hybrid orbital decreases from
50% to 25% and p-character increases from 50% to 75% and in any hybrid orbital total % of s &
p- character remains 100%. By increasing p-character the hybrid orbitals become elongated
hence, their overlapping extent decreases that is results into weak bond energy also bond angle
decreases.

20. Statement-1 : On decreasing s-character in hybrid orbitals, bond angle decreases.


Statement-2 : p-orbitals are at 90° to one another.
(A) Statement-1 is true, statement-2 is true and statement-2 is correct explanation for
statement-1.
(B) Statement-1 is true, statement-2 is true and statement-2 is NOT the correct explanation for
statement-1.
(C) Statement-1 is true, statement-2 is false.
(D) Statement-1 is false, statement-2 is true.

21. The type of overlapping which produces bond of maximum bond energy is:
(A) sp3 – 1s (B) sp2 – 1s
(C) sp –1s (D) All have same bond energy

22. The molecule having bond pair in which % of s & p character is different from rest bond pairs.
(A) SiF4 (B) BCl3
(C) CF3H (D) CCl4

Nothing is impossible, the word itself says ‘I’m possible’!


78 CHEMICAL BONDING

MORE THAN ONE MAY BE CORRECT

23. Which of the following option is/are correct for Fe


26

(A) Number of electrons having l + m value equal to zero in 26 Fe may be 14


(B) Number of electrons having n + l value equal to zero in Fe may be 0
26

(C) Number of electrons having l + m value equal to zero in 26 Fe may be 0


(D) The magnetic quantum number define the orientation of an electron

24. Which of the following have fractional bond order?


(A) C2 (B) O2 (C) NO (D) NO 

25. Which of the following statement is/are correct


(A) the peroxide ion has a bond order of 1 while the oxygen molecule has a bond order of 2
(B) the peroxide ion has a weaker bond than the dioxygen molecule
(C) the peroxide ion as well as the dioxygen molecule are paramagnetic
(D) the bond length of the peroxide ion is greater than that of the dioxygen molecule

26. Which is/are true about OF2 & Cl2 O


(A) Both are sp3 hybridized (B) Bond angle in OF2 is less than 10928
(C) Bond angle in Cl2 O is greater than 10928
(D) Bond angle in both the cases are 10928

27. Which of the following options is/are correct ?


(A) ClF3 and SF4 follow the bent rule and lone pair of electron goes to the equatorial position
(B) The bond angle of H2S is lower than NH3
(C) D2 O sinks in water , but ice floats on water
(D) In PCl5 , all the P – Cl bond length are equal

28. Which of the following statements about alkaline earth metals are correct ?
(A) Hydration energy of Be 2  is greater than that of Sr 2 
(B) MgCl2 is more covalent than CaCl2
(C) Ba  OH2 is stronger base than Mg  OH2

(D) Hydration energy of Sr 2  is greater than that of Be 2 

29. Element A & B has five electrons in their valence shell but their penta halides are not possible.
Hydride of A is stable but B is not. Which of the following option is/are correct for element A
and element B :
(A) Element A is N and Element B is Bi
(B) Hydride of element A is NH3 and hydride of element B is BiH3
(C) Due to inert pair effect in element B lower oxidation state is stable
(D) The hybridisation in their respective hydrides are sp2 & sp2

CatalyseR Eduventures (India) Pvt. Ltd.


CHEMICAL BONDING 79

30. Which of the following statement(s) is/are correct?


(A) For the covalent bond formation the minimum requirement of electrons is one.
(B) In the formation of H2 , B.M.O. contains one electron.
(C) In the formation of H2 , A.B.M.O. contains one electron.
(D) The bond order of H2 is 0.5.

31. The difference in bond angles in tetrahedral structures of different compounds is due to
(A) The shape of the molecule
(B) The presence of unshared lone pair of electrons
(C) The difference in electro negativity
(D) The presence of bond pair of electrons

32. Which of the following species contain coordinate covalent bond :


(A) AlCl3 (B) CO (C) [Fe(CN)6 ]4 (D) N3

33. There is change in the hybridisation when:


(A) NH3 combines with H (B) AlH3 combines with H
(C) NH3 forms NH2 (D) SiF4 forms SiF62

34. Which of the following statement is/are correct


(A) Hybridisation is the mixing of pure atomic orbitals having less energy difference
(B) sp3d2  hybrid orbitals are at 90º to each other
(C) sp3 d  hybrid orbitals are directed towards the corners of a regular tetrahedron
(D) sp3d2  hybrid orbitals are directed towards the corners of a regular octahedron

35. Which of the following statements are correct?


(A) The crystal lattice of ice is formed by covalent as well as hydrogen bonds
(B) The density of water increases when heated from 0º C to 4º C
(C) Above 4º C the thermal agitation of water molecules increases. Therefore, intermolecular
distance increases and water starts expanding
(D) The density of water decreases from 0º C to a maximum at 4º C

36. For propadiene H2 C  C  CH2 , correct statement(s) is / are :


1 2 3
(A) Molecule is non planar
(B) Molecule is nonpolar
(C) Nodal plane of p-bond formed by C1 & C2 is perpendicular to that of formed by C2 & C3 .
(D) Nodal plane of p-bond formed by C1 & C2 is coplanar with that of formed by C2 & C3 .

37. Molecule(s) having both polar and non polar bonds is / are
(A) O 2F2 (B) S2 Cl2
(C) N2H4 (D) S2F10

Nothing is impossible, the word itself says ‘I’m possible’!


80 CHEMICAL BONDING

38.

(A) has intermolecular H - bonding (B) has intramolecular H- bonding


(C) has low boiling point (D) is steam-volatile

39. Which of the following factors are responsible for van der Waals forces ?
(A) Instantaneous dipole-induced dipole interaction
(B) Dipole-induced dipole interaction and ion-induced dipole interaction
(C) Dipole-dipole interaction and ion-induced dipole interaction
(D) Small size of molecule

40. Which of the following molecules have intermolecular hydrogen bonds ?


(A) KH2PO 4 (B) H3BO3 (C) C6H5 CO 2H (D) CH3 OH

41. Which of the following have dipole moment ?


(A) nitrobenzene (B) p-chloronitrobenzene
(C) m-dichlorobenzene (D) o-dichlorobenzene

MATCH THE COLUMN

42. List I contains compounds which on heating gives the compounds in List II
List I List II
(A) (NH4)2CO3 (p) N2
(B) Ca(NO3)2 (q) N2O
(C) (NH4)2Cr2O7 (r) NO2
(D) NH4NO3 (s) NH3

43. Match the following columns


List I List II
(A) NaCl (p) Soluble in water
(B) AgCl (q) Lattice energy > Hydration energy
(C) CdCl2 (r) Lattice energy < Hydration energy
(D) FeSO4 (s) Cation has Pseudo inert (18 e– in
valence shell) gas configuration

44. Match the following


List I List II
(A) H3PO4 (p) on heating gives meta (acid or salt) product
(B) H3BO3 (q) dibasic in nature
(C) Microcosmic salt (r) monobasic in nature
(D) NaH2PO4 (s) anion is sp3 hybridised

CatalyseR Eduventures (India) Pvt. Ltd.


CHEMICAL BONDING 81

45. Match List I with List II and select the correct answer .
List I List II
+
(A) NO2 (p) 180º
(B) NO2 (q) 132º

(C) NO2 (r) 120º
(D) NO3– (s) 115º
(t) 109º

46. Match the Column - I with Column - II. Note that Column - I may have more than one matching
options in Column - II.
Column-I Column-II
(A) Hypo phosphoric acid (p) All hydrogen are ionizable in water
(B) Pyro phosphorous acid (q) Lewis acid in water
(C) Boric acid (r) Monobasic in water
(D) Hypo phosphorous acid (s) sp3 hybridised central atom

47. Match the column-


List I List II
(A) BrF3 (p) One angle  90°

(B) TeF5¯ (q) Central atom is sp3d2 hybridised


(C) IF7 (r) Non planar
(D) XeF4 (s) Polar

48. Column I Column II

(A) I (CN)2¯ (p) Having p  p bond & D  0


(B) CO32– (q) Having p  d bond & D  0
(C) XeO2F2 (r) Planar
(D) SOF4 (s) Central atom is sp3d

49. Column I Column II

(A) H2S2O5 (p) Central atom is sp3 hybridised

(B) H6B2O72– (q) M–O–M ie oxo linkage is present

(C) H4P2O6 (r) M–M ie oxo linkage is absent

(D) H6Si2O7 (s) Non planar

Nothing is impossible, the word itself says ‘I’m possible’!


82 CHEMICAL BONDING

INTEGER ANSWER TYPE QUESTION


50. How many of the following compounds have significant involvement of d orbital in  bonding?
H3PO2 ,BCl3 ,H2S 4 O6 , XeO 4 , XeOF4 ,Fe  CO 5 ,SF6 ,NH4 Cl,ClO 2 ,NO2

51. How many of the following statements are correct?


(i) Among sp3 d,sp3 d2 and sp3 d3 hybridisation, the maximum number of 90 angles between
bond pair-bond pair of electrons is observed in sp3 d2 hybridisation
(ii) Among SF4 ,CF4 and XeF4 , interatomic bond angle of 10928' is observed only in SF4
(iii) Among N2F4 ,N2F2 and N2 , the largest N-N bond length is found in N2F4 .

(iv) Among NF3 ,NO3 ,BF3 ,H3 O  and NH3 , NF3 ,H3 O   and BF3 ,NO3  are isostructural pairs.
(v) Among NH3 ,PH3 , AsH3 and SbH3 ,SbH3 has smallest bond angle.
(vi) Among SbCl3 ,SbBr3 and Sbl3 ,Sbl3 has the largest bond angle.

52. The bond order of the underlined species : NOHSO 4 is.

53. How many maximum atoms lie in the same plane in the structure of methylenesulphurtetrafluoride
 CH2SF4  ?
54. There will be three different fluorine-fluorine distances in molecule CF2  C 2 CF2 . Assuming ideal
bond angles for a particular hybridisation (assume no distortion due to double bonds) find out the
smaller fluorine-flourine distance and fill the result (in pm) in the increasing order in your answer
sheet. Round off your answer to the nearest whole number.
(Given that C-F bond length = 134 pm, C  C bond length  134pm 3  1.73 )

55. The difference in the number of  and  bonds in trimer of SO3 i.e. S3 O9 is :
(Consider no co-ordinate bond to be present)

56. In how many following species, the central atoms have two lone pairs of electrons?
XeF4 XeF5 F2 SeO2
XeF3 XeOF4 ClOF3
ICl4 SCl2 OSF4

57. BrF3 is a liquid which considerably undergoes ionization to form cationic and anionic species.
Based on VSEPR theory, number of 90 degree F  Br  F bond angles is …………… in anionic
species.

 
 BrF2   BrF4 
2BrF3 

58. Sum of antibonding  electrons  * electrons  in species O2 ,O2 and O22 are.

59. Amongst the following, the total number of species which does/do not exist is:
SF6 ,BF63 ,SF4 ,OF4 ,AlF63 ,PH5 ,PCl5 ,NCl5 ,SCl6

CatalyseR Eduventures (India) Pvt. Ltd.


CHEMICAL BONDING 83

60. How many oxides are soluble in moderately concentrated aqueous solution of NaOH.

SO3 Cl2 O 7 N2 O5 CO K 2O Cr2 O 3 BaO GeO 2


(1) (2) (3) (4) (5) (6) (7) (8)



Nothing is impossible, the word itself says ‘I’m possible’!


84 CHEMICAL BONDING

EXERCISE # 04 SUBJECTIVE EXERCISE (LEVEL # 01)



1. In solid Na Cl one Na  is surrounded by six Cl ions find out total number of directional bonds
formed by each Na  with Cl ions.

2. Find out total number of p  d bonds present in any one of its structures in case of PO 43 

3. Find out total number of p  p bonds present in ClO3  .

4. How many number of atomic orbitals are required, so that their mixing produces hybrid orbitals
each having % of s-character equal to 17%.

5. AgNO 3 gives a white precipitate with NaCl but not with CCl4 . Why?

6. What should be the structure shape of the following as per VSEPR theory?
(a) XeF2 (b) XeF4 (c) PBr5 (d) OF2 (e) I3 and (f) I3

7. The percent ionic character in HCl is 18.08. The observed dipole moment is 1.08 D. Find the
inter-nuclear distance in HCl.

8. Assuming that all the four valency of carbon atom in propane pointing towards the corners of a
regular tetrahedron. Calculate the distance between the terminal carbon atoms in propane.
Given, C  C single bond length is 1.54 Å.

9. The dipole moment of HBr is 7.95 debye and the intermolecular separation is 1.94  10 10 m . Find
the % ionic character in HBr molecule.

10. Dipole moment of LiF was experimentally determined and was found to be 6.32 D. Calculate
percentage ionic character in LiF molecule Li  F bond length is 156 pm.

11. A diatomic molecule has a dipole moment of 1.2 D. If bond length is 1.0 Å, what percentage of an
electronic charge exists on each atom.

12. Which will have a higher boiling point, Br2 or ICl, & why?

13. Arrange noble gases, in the increasing order of b.p.

14. Explain the molecule of magnesium chloride is linear whereas that of stannous chloride is
angular.

15. Give reason carbon oxygen bond lengths in formic acid are 1.23 A° & 1.36 A° and both
the carbon oxygen bonds in sodium formate have the same value i.e. 1.27 A°.

16. Explain the first I.E. of carbon atom is greater than that of boron atom whereas the reverse
is true for the second I.E.

17. Explain why the dipolemoment of NH3 is more than that of NF3.

CatalyseR Eduventures (India) Pvt. Ltd.


CHEMICAL BONDING 85

18. The experimentally determined N - F bond length in NF3 is greater than the sum of single
bond covalent radii of N & F . Explain.

19. Explain the difference in the nature of bonding in LiF & LiI.

20. Explain why o-hydroxybenzaldehyde is a liquid at room temperature, while p-


hydroxybenzaldehyde is a high melting solid.

21. What are the types of bond present in B 2H6 ?

22. Arrange toluene, m–dichlorobenzene, o–dicholorobenzene and p–dichlorobenzene in order of


increasing dipole moment.

23. Interpret the non-linear shape of H2S molecule & non planar shape of PCl3 using VSEPR
theory.

24. Discuss the hybridisation of C - atoms in allene  C3H4  and show the p - orbital overlaps.

25. In the hydrides of group VI elements the central atoms involve sp3 hybridisation but the bond
angles decrease in the order, H2 O, H2 S, H2 Si, H2 Te . How would you account for this?



Nothing is impossible, the word itself says ‘I’m possible’!


86 CHEMICAL BONDING

EXERCISE # 05 SUBJECTIVE EXERCISE (LEVEL # 02)


1. a) KHF2 is known but KHCl2 is unknown. Why?
b) Ice is lighter than water. Why?

2. a) LiF has the lowest solubility of the Group - I metal halides why?
b) AgI is least soluble among silver halides explain.

3. a) Me3N is pyramidal in shape while (SiH3)3N is planar. Why?


b) NF3 is stable while NCl3 and NI3 are explosive - Explain.

4. a) Nitrogen does not form NCl5 or NF5 but phosphorous can. Explain.
b) Valency of oxygen is generally two whereas sulfur shows valencies of two, four & six. Why?

5. Boron does not usually form a cation. Why?

6. PCl5 exists in the solid in the form of [PCl4]+ [PCl6]–, yet it is a non conductor of electricity. Why?

7. a) NF3 does not undergo hydrolysis while NCl3 undergoes hydrolysis. Explain
b) SF4 is instantly hydrolysed but SF6 is stable towards hydrolysis. Why?

8. a) Compare the dipole moments of CH3Cl, CH2Cl2, CHCl3, CCl4.


b) Compare the dipole moments of H2O and F2O.

9. a) F2O has got a bond angle of 103 but Cl2O has got a angle of 110. Explain
b) Bond angle of H2O is 104 while that of H2S in 92. Explain

10. In some polar solvents PCl5undergoes an ionisation reaction in which Cl– ion leave one PCl5
molecule and attaches itself to another
2PCl5 PCl4+ + PCl6–
What are the changes in geometrical shapes that occur in this ionisation?

11. a) LiCl is hydrated, but NaCl is always anhydrous. Why?


b) Why is it that although Li+ is far smaller than the other metals ions, it moves through a
solution less rapidly than the others?

12. a) Compare the bond energies of O2, O2 & O2 .


b) Compare the bond energies of N2, N 2 & N 2 .

13. Bond energy of H – H, F – F and H – F bonds are 104, 38 and 135 kcal mol–1 respectively.
Calculate resonance energy in H – F molecule.

14. A molecular compound is composed of 60.4% of Xe, 22.1% of O and 17.5% of F by mass. If the
molecular weight is 217.3 amu, what is the molecular formula? Predict the molecular geometry
using VSEPR model.

CatalyseR Eduventures (India) Pvt. Ltd.


CHEMICAL BONDING 87

15. 2Li + O2  Li2O


2Na + O2  Na2O2
Why the number of oxygen atoms combining with each metal increases?
2+ +
16. In analytical chemistry Hg is identified by reduction of Hg . However the actual structure of the
latter is found to be the dimeric species Hg22+ which is more stable than the monomer Hg+. Why
is it so?

17. Give MO configuration and bond orders of H2, H2–, He2 and He2–. Which species among the
above are expected to have same stabilities?

18. CsCl is formed according to the following equation


1
Cs(s) + Cl2(g)  CsCl(s)
2
The enthalpy of sublimation of Cs, enthalpy of dissociation of chlorine, ionization energy of Cs
and electron affinity of chlorine are 81.2, 243.0, 375.7 and – 348.3 kJ mol–1 respectively. The
energy change involved in the formation of CsCl is – 388.6 kJ mol–1. Calculate the lattice energy
of CsCl.

19. Set up Born Haber cycle for determination of solvation of Mg2 ions in eV/atom by water given the
data.
Enthalpy of atomization of Mg = 167.2 kJ mol1
First ionization energy of Mg = 7.646 eV
2nd ionization energy of Mg = 15.035 eV
Enthalpy of dissocation of Cl2 = 241.6 kJ kJ mol–1
E.A. of Chlorine = – 3.78 eV
Enthalpy of formation of MgCl2(s) = – 639.5 kJ mol–1
Enthalpy of solution of MgCl2(s) = – 150.5 kJ mol–1
Enthalpy of hydration of Cl–(g) = – 383.7 kJ mol–1



























































Nothing is impossible, the word itself says ‘I’m possible’!


88 CHEMICAL BONDING

EXERCISE # 06 KVPY, OLYMPIADS CORNER


1. The numbers of lone pairs and bond pairs in hydrazine are, respectively [KVPY 2014]
(A) 2 and 4 (B) 2 and 6 (C) 2 and 5 (D) 1 and 5
2. The number of C-C sigma bonds in the compound
O

is [KVPY 2014]

(A) 16 (B) 17 (C) 18 (D) 11


3. The diamagnetic species is [KVPY 2014]
(A) NO (B) NO2 (C) O2 (D) CO2

4. The element which readily forms an ionic bond has the electronic configuration [KVPY 2015]
2 2 2 2 2 1 2 2 2
(A) 1s 2s 2p (B) 1s 2s 2p (C) 1s 2s 2p (D) 1s2 2s2 2p6 3s1

5. The molecule having the highest dipole moment is : [NSEC 2014]


(A) CO2 (B) CH4 (C) NH3 (D) NF3

6. The species which has triangular planar geometry is [NSEC 2014]



(A) NF3 (B) NO 3 (C) AlCl3 (D) SbH3

7. The correct order of stability for the following species is [NSEC 2014]
   
(A) Li2  He  O  C2
2 2 (B) C2  O  Li2  He
2 2

(C) He2  Li2  C2  O2 (D) O2  C2  Li2  He2

8. The correct order of dipole moment for the following molecules is : [NSEC 2014]
F F F F F F

F F F
F
I II III IV
(A) IV > I > III > II (B) I > IV > III > II
(C) III > I > II > IV (D) II > III > IV > I
9. For SF4 , the molecular geometry and hybridization of the central atom respectively are
[NSEC 2014]
(A) Square planar, dsp2 (B) Tetrahedral, sp3
(C) Seesaw, sp3 d (D) Square pyramid, sp3 d

CatalyseR Eduventures (India) Pvt. Ltd.


CHEMICAL BONDING 89

10. The species having highest bond energy is [NSEC 2015]

(A) O2 (B) O2 (C) O2 (D) O22

11. The structure of molecular of N  SiMe3 3 is : [NSEC 2015]

(A) Pyramidal with angle close to 110


(B) T-shaped with angle 90
(C) Bent T-shaped with angle close to 89
(D) Trigonal planar with bond angle close to 120

12. The order of p  d interaction in the compounds containing bond between Si / P / S / Cl and
oxygen is in the order [NSEC 2015]
(A) P  Si  Cl  S (B) Si  P  S  Cl
(C) S  Cl  P  Si (D) Si  P  S  Cl

13. From the given structures, the correct structure(s) of PF3 Cl2 is/are [NSEC 2015]
F Cl Cl
Cl F F
(I) P F (II) P F (III) P F
Cl F Cl
F Cl F
(A) Only I (B) Only II (C) Only III (D) I, II and III



Nothing is impossible, the word itself says ‘I’m possible’!


90 CHEMICAL BONDING

EXERCISE # 07 JEE (MAIN) CORNER


1. Which of the following molecules/ions does not contain unpaired electrons? [2006]
2 
(A) O 2 (B) B2 (C) N2 (D) O2

2. A metal, M forms chlorides in + 2 and + 4 oxidation states. Which of the following statements
about these chlorides is correct? [2006]
(A) MCl2 is more volatile than MCl4
(B) MCl2 is more soluble in anhydrous ethanol than MCl4
(C) MCl2 is more ionic than MCl4 (D) MCl2 is more easily hydrolysed than MCl4

3. In which of the following molecules/ions are all the bonds not equal? [2006]

(A) SF4 (B) SiF4 (C) XeF4 (D) BF 4

4. Which of the following hydrogen bonds is the strongest ? [2007]


(A) O – H ... O (B) O – H ... F (C) F – H ... H (D) F – H ... F

5. The charge/ size ratio of a cation determines its polarizing power. Which one of the following
sequences represents the increasing order of the polarizing power of these cationic species,
K  ,Ca2 ,Mg2 ,Be2 ? [2007]
(A) K   Ca2  Mg2  Be2 (B) Ca2  Mg2  Be2  K 
(C) Mg2  Be2  K   Ca2 (D) Be2  K   Ca2  Mg2

6. In which of the following ionization processes, the bond order has increased and the magnetic
behaviour has changed ? [2007]
(A)  O2
O2  (B) N2
N2  (C)  C2
C2  (D) NO  NO 

7. Which one of the following pairs of species have the same bond order? [2008]
     
(A) CN and CN (B) O2 and CN (C) NO and CN (D) CN and NO 

8. The bond dissociation energy of B – F in BF3 is 646kJmol1 whereas that of C – F in CF4 is


515kJmol1. The correct reason for higher B – F bond dissociation energy as compared to that of
C – F is : [2009]
(A) Stronger  bond between B and F in BF3 as compared to that between C and F in CF4 .
(B) Significant p  p interaction between B and F in BF3 whereas there is no possibility of
such interaction between C and F in CF4 .
(C) Lower degree of p - p interaction between B and F in BF3 than that between C and F in
CF4 .
(D) Smaller size of B - atom as compared to that of C - atom.

9. Using MO theory predict which of the following species has the shortest bond length? [2009]
(A) O 2+ (B) O 2– (C) O22– (D) O22+

CatalyseR Eduventures (India) Pvt. Ltd.


CHEMICAL BONDING 91

10. Ortho-Nitrophenol is less soluble in water than p- and m- nNitrophenols because [2012]
(A) Melting point of o-Nitrophenol is lower than those of m- and p-isomers.
(B) o-Nitrophenol is more volatile in steam than m- and p-isomers.
(C) o-Nitrophenol shows intramolecular H-bonding.
(D) o-Nitrophenol shows intermolecular H-bonding.

11. Which one of the following molecules is expected to exhibit diamagnetic behaviour? [2013]
(A) C2 (B) N2 (C) O2 (D) S2

12. Which of the following arrangements does not represent the correct order of the property stated
against it? [2013]
(A) V 2   Cr 2  Mn2   Fe 2  : paramagnetic behaviour
(B) Ni2   Co 2   Fe 2   Mn2  : ionic size
(C) Co 3   Fe3   Cr 3   Sc 3  : stability in aqueous solution
(D) Sc  Ti  Cr  Mn : number of oxidation states

13. Which of the following is the wrong statement? [2013]



(A) ONCl and ONO are not isoelectronic. (B) O 3 molecule is bent
(C) Ozone is violet-black in solid state (D) Ozone is diamagnetic gas.

14. In which of the following pairs of molecules/ions, both the species are not likely to exist?
[2013]
 2  2 2  2
(A) H , He
2 2 (B) H , He
2 2 (C) H , He2
2 (D) H , He
2 2

15. Stability of the species Li2 , Li2 and Li2 increases in the order of : [2013]
(A) Li2  Li2  Li2 (B) Li2  Li2  Li2 (C) Li2  Li2  Li2 (D) Li2  Li2  Li2

16. Which one of the following properties is not shown by NO? [2014]
(A) It’s bond order is 2.5 (B) It is diamagnetic in gaseous state
(C) It is a neutral oxide
(D) It combines with oxygen to form nitrogen dioxide

17. For which of the following molecule significant   0? [2014]


Cl CN OH SH

(1) Cl (2) CN (3) OH (4) SH

(A) Only (3) (B) (3) & (4)


(C) Only (1) (D) (1) & (2)

18. The species in which the N atom is in a state of sp hybridization is : [2016]


  
(A) NO 2 (B) NO 3 (C) NO2 (D) NO 2

Nothing is impossible, the word itself says ‘I’m possible’!


92 CHEMICAL BONDING

19. Which of the following species is not paramagnetic? [2017]


(A) CO (B) O2 (C) B2 (D) NO



CatalyseR Eduventures (India) Pvt. Ltd.


CHEMICAL BONDING 93

EXERCISE # 08 JEE (ADVANCED) CORNER


1. The percentage of p-character in the orbitals forming P–P bonds in P4 is [2007]
(A) 25 (B) 33 (C) 50 (D) 75
2. The structure of XeO3 is [2007]
(A) linear (B) planar (C) pyramidal (D) T-shaped
3. The species having bond order different from that in CO is [2007]
  
(A) NO (B) NO (C) CN (D) N2
4. Statement-1 : p-Hydroxybenzoic acid has a lower boiling point than o-hydroxybenzoic acid.
because
Statement-2 : o-Hydroxybenzoic acid has intramolecular hydrogen bonding. [2007]
(A) Statement-1 is True, Statement-2 is True; Statement-2 is a correct explanation for
Statement-1.
(B) Statement-1 is True, Statement-2 is True; Statement-2 is NOT a correct explanation for
Statement-1.
(C) Statement-1 is True, Statement-2 is False.
(D) Statement-1 is False, Statement-2 is True.
5. Statement-1 : In water, orthoboric acid behaves as a weak monobasic acid.
because
Statement-2 : In water, orthoboric acid acts as a proton donor. [2007]
(A) Statement-1 is True, Statement-2 is True; Statement-2 is a correct explanation for
Statement-1.
(B) Statement-1 is True, Statement-2 is True; Statement-2 is NOT a correct explanation for
Statement-1.
(C) Statement-1 is True, Statement-2 is False
(D) Statement-1 is False, Statement-2 is True
6. The nitrogen oxide(s) that contain(s) N–N bond(s) is/are [2009]
(A) N2O (B) N2O3 (C) N2O4 (D) N2O5

7. The correct stability order of the following resonance structures is [2009]


   
H2 C  N N H2 C N  N H2 C N  N
(I) (II) (III)
(A) (I) > (II) > (III) (B) (I) > (III) > (II) (C) (II) > (I) > (III) (D) (III) > (I) > (II)

8. Column – I Column – II [2009]


(A) B 2 (p) Paramagnetic

(B) N2 (q) Undergoes oxidation



(C) O 2 (r) Undergoes reduction
(D) O2 (s) Bond order  2
(t) Mixing of s and p-orbitals

Nothing is impossible, the word itself says ‘I’m possible’!


94 CHEMICAL BONDING

9. The number of water molecule(s) directly bonded to the metal centre in CuSO4 .5 H2 O is
[2009]

10. Based on VSEPR theory, the number of 90 degree F–B–F angles in BrF5 is [2010]

11. The species having pyramidal shape is [2010]


(A) SO3 (B) BrF3
(C) SiO23 (D) OSF2

12. Assuming that Hund’s rule is violated, the bond order and magnetic nature of diatomic molecule
B 2 is [2010]
(A) 1 and diamagnetic
(B) 0 and diamagnetic
(C) 1 and paramagnetic
(D) 0 and Paramagnetic

13. The correct statement(s) about O3 is(are) [2013]


(A) O–O bond lengths are equal.
(B) Thermal decomposition of O3 is endothermic.
(C) O3 is diamagnetic in nature.
(D) O3 has a bent structure.

14. EDTA4is ethylenediaminetetraacetate ion. The total number of N – Co – O bond angles in

[Co(EDTA)]1complex ion is [2013]

15. The total number of lone-pairs of electrons in melamine is [2013]

16. Hydrogen bonding plays a central role in the following phenomena : [2014]
(A) Ice floats in water
(B) Higher Lewis basicity of primary amines than tertiary amines in aqueous solutions
(C) Formic acid is more acidic than acetic acid
(D) Dimerisation of acetic acid in benzene.
17. Assuming 2s-2p mixing is NOT operative, the paramagnetic species among the following is
[2014]
(A) Be2 (B) B2
(C) C2 (D) N2

18. A list of species having the formula XZ 4 is given below. [2014]


2 2 2 2
XeF4 ,SF4 ,SiF4 ,BF4 , Cu NH3 4  , FeCl4  ,CoCl4  and PtCl4  .
Defining shape on the basis of the location of X and Z atoms, the total number of species having
a square planar shape is

CatalyseR Eduventures (India) Pvt. Ltd.


CHEMICAL BONDING 95

19. Match the orbital overlap figures shown in Column – I with the description given in Column – II
and select the correct answer using the code given below the lists. [2014]
Column – I Column – II

(A) 1. p  d  antibonding

(B) 2. d  d  bonding

(C) 3. p  d  bonding

(D) 4. d  d  antibonding

Codes :
P Q R S
(A) 2 1 3 4
(B) 4 3 1 2
(C) 2 3 1 4
(D) 4 1 3 2

20. Among the triatomic molecules/ions, BeCl2 ,N3 ,N2O,NO2 ,O3 ,SCl2 ,ICl2 ,I3 and XeF2 , the total
number of linear molecule(s)/ion(s) where the hybridization of the central atom does not have
contribution from the d-orbital(s) is
[Atomic Number : S = 16, Cl = 17, I = 53 and Xe = 54] [2015]

21. The colour of the X 2 molecules of group 17 elements changes gradually from yellow to violet
down the group. This is due to : [2017]
(A) Decrease in ionization energy down the group
(B) Decrease in HOMO-LUMO gap down the group
(C) Decrease in  *  * gap down the group
(D) the physical state of X 2 at room temperature changes from gas to solid down the group.

22. The sum of the number of lone pair of electrons on each central atom in the following species is
2 2 
TeBr6  ,  BrF2  , SNF3 and  XeF3 
(Atomic numbers : N = 7, F = 9, S = 16, Br = 35, Te = 52, Xe = 54) [2017]

Nothing is impossible, the word itself says ‘I’m possible’!


96 CHEMICAL BONDING

 
23. Among H 2 , He2 , LI 2 , Be2 , B2 , C2 , N 2 , O2 and F2 , the number of diamagnetic species is
………………… .
(Atomic numbers : H = 1, He = 2, Li = 3, Be = 4, B = 5, C = 6, N = 7, O = 8, F = 9) [2017]

24. Among the following, the correct statement(s) is (are) : [2017]


(A) BH 3 has the three-centre two-electron bonds in its dimeric structure
(B) Al  CH 3 3 has the three-centre two-electron bonds in its dimeric structure
(C) AlCl3 has the three-centre two-electron bonds in its dimeric structure
(D) The Lewis acidity of BCl3 is greater than that of AlCl3



CatalyseR Eduventures (India) Pvt. Ltd.


CHEMICAL BONDING 97

ANSWER KEYS
EXERCISE # 01 OBJECTIVE EXERCISE (LEVEL # 01)

1 2 3 4 5 6 7 8 9 10
C B D D A C C D B D
11 12 13 14 15 16 17 18 19 20
C D D C A B B A C A
21 22 23 24 25 26 27 28 29 30
D A A C C C B B C D
31 32 33 34 35 36 37 38 39 40
B B A B A C D D C C
41 42 43 44 45 46 47 48 49 50
D A B B C A C A B C
51
A

EXERCISE # 02 OBJECTIVE EXERCISE (LEVEL # 02)

1 2 3 4 5 6 7 8 9 10
B C D B D D A C C D
11 12 13 14 15 16 17 18 19 20
A D B D C D A D C D
21 22 23 24 25 26 27 28 29 30
C B B C A C C D A A
31 32 33 34 35 36 37 38 39 40
B A C C C C B D A A
41 42 43 44 45 46 47 48 49 50
B C A C D D A B A B
51 52 53 54 55 56 57 58 59 60
B A D D C D C B D D
61 62 63
C A D

Nothing is impossible, the word itself says ‘I’m possible’!


98 CHEMICAL BONDING

EXERCISE # 03 OBJECTIVE EXERCISE (LEVEL # 03)

1 2 3 4 5 6 7 8 9 10
A D A D C C A B B A
11 12 13 14 15 16 17 18 19 20
A C C C C D C A D B
21 22 23 24 25 26 27 28 29 30
C C ABD ABC ABD ABC ABC ABC ABC ABD
31 32 33 34 35 36 37 38 39 40
BCD BCD BD ABD ABC ABC ABCD BCD ABC ABCD
41 42 43
ABCD (A) – s, (B) – r, (C) – p, (D) - q (A) – pr, (B) – qs, (C) – prs, (D) – pr
44 45
(A) – ps, (B) – pr, (C) – prs, (D) - pqs (A) – p, (B) – q, (C) – s, (D) - r
46 47
(A) – ps, (B) – s, (C) – qr, (D) - rs (A) – ps, (B) – pqrs, (C) – pr, (D) - pq
48 49 50
(A) – prs, (B) – pr, (C) – qs, (D) - qs (A) – prs, (B) – pqs, (C) – prs, (D) - pqs 6
51 52 53 54 55 56 57 58 59 60
5 3 6 232 6 5 4 9 5 7

EXERCISE # 04 SUBJECTIVE EXERCISE (LEVEL # 01)


Q.1 zero Q.2 1 Q.3 0 Q.4 6
Q.5 NaCl ionic compound which dissociate to give Cl¯ ions where as CCl4 can not ionised to give
Cl¯.
Q.6 (a) Linear, (b) square planar, (c) T.B.P. (d) bent, (e) linear, (f) bent
Q.7 1.2Å Q.8 2.514 Å Q.9 85%
Q.10 84.5% Q.11 25% Q.12 ICl
Q.13 He < Ne < Ar < Kr < Xe < Rn
Q.14 Lone pair Q.15 Resonance
Q.17 Lone pair contribution Q.19 LiF  Ionic charge, LiI  covalent charge
Q.20 Intra-H-bonding in o-hydroxybenzaldehyde
Q.24
CH 3  C  CH 3
  
sp 2 sp sp2

CatalyseR Eduventures (India) Pvt. Ltd.


CHEMICAL BONDING 99

EXERCISE # 06 KVPY, OLYMPIADS CORNER

1 2 3 4 5 6 7 8 9 10
C B D D D B C B C B
11 12 13
D B A

EXERCISE # 07 JEE (MAIN) CORNER

1 2 3 4 5 6 7 8 9 10
A C A D A D D B D C
11 12 13 14 15 16 17 18 19
AB A ABCD C B B B D A

EXERCISE # 08 JEE (ADVANCED) CORNER

1 2 3 4 5 6 7 8 9 10
D C A D C ABC B 4 .0
11 12 13 14 15 16 17 18 19 20
D A ACD 8 6 ABD C 4 C 4
21 22 23 24
BC 6 6 ABD

8. A-p, r, t ; B-s, t ; C-p, q ; D-p, q, s

Nothing is impossible, the word itself says ‘I’m possible’!

You might also like